Category Archives: Chuyên đề

PHƯƠNG TRÌNH NGHIỆM NGUYÊN DẠNG LUỸ THỪA

A. MỘT SỐ CHÚ Ý KHI GIẢI PHƯƠNG TRÌNH DẠNG LŨY THỪA
Nhận xét: Để giải phương trình nghiệm nguyên dạng lũy thừa ta chú ý một số phương pháp thường sử dụng

  • Sử dụng đồng dư để xét tính chẵn lẻ, hay modun của nghiệm.
  • Phân tích thành thừa số.
  • Đánh giá bất đẳng thức.

Do sử dụng nhiều đồng dư, do đó ta chú ý một số tính chất về đồng dư sau Tính chất 3.2. Cho $a$ là một số nguyên tùy ý. Khi đó
(a) $a^2 \equiv 0,1(b\mod 3)$;
(b) $a^2 \equiv 0,1(b\mod 4)$
(c) $a^2 \equiv 0,1,4 (b\mod 8)$;
(d) $a^2 \equiv 0,1,4 (b\mod 5)$;
(e) $a^3 \equiv-1,0,1 (b\mod 7)$
(f) $a^3 \equiv-1,0,1(b\mod 9)$.

Tính chất 3.3. Cho $p$ là một số nguyên tố và $a, b, c, n$ là các số nguyên dương. Ta có
(a) $a^n \vdots p \Leftrightarrow a \vdots p$;
(b) Nếu $a b=p^n$ thì $\left\{\begin{array}{l}a=p^k \\\ b=p^{n-k}\end{array} \quad\right.$ với $k \in \mathbb{N}$ thỏa $0 \leq k \leq n$;
(c) Nếu a b=c^n và (a, b)=1 thì $a=s^n \text { và } b=r^n$ với $s, r \in \mathbb{N}$.

B MỘT SỐ VÍ DỤ
Ví dụ 3.29. Tìm các số nguyên $x, y$ thỏa mān $x^3+1=4 y^2$.

Hướng dẫn giải

Giả sử tồn tại các số nguyên $x, y$ thỏa mãn $x^3+1=4 y^2$. Ta có
$$
x^3=4 y^2-1=(2 y-1)(2 y+1) \text {. }
$$

Đặt $d=(2 y-1,2 y+1)$, ta có $d$ lẻ và $\left\{\begin{array}{l}d \mid 2 y-1 \\\ d \mid 2 y+1\end{array}\right.$.
Do đó $d \mid 2$, suy ra $d=1$ (vì $d$ lẻ). Như vậy $2 y-1$ và $2 y+1$ nguyên tố cùng nhau.
Kết hợp với (3.1) ta suy ra $2 y-1=a^3$ và $2 y+1=b^3$ với $a, b \in \mathbb{Z}$.
Dẫn đến $b^3-a^3=2$ hay $(b-a)\left(b^2+b a+a^2\right)=2$. Từ đó ta được $b=1$ và $a=-1$, suy ra $y=0$ và khi đó $x=-1$. Thử lại thỏa.
Vậy $(x, y)=(-1,0)$.

Ví dụ 3.30. Giải phương trình nghiệm nguyên $x^5+2023 x=5^y+2$.

Hướng dẫn giải

Giả sử tồn tại các số nguyên $x, y$ thỏa mãn $x^5+2023 x=5^y+2$.
Vì $5^y+2$ lẻ nên $x$ lẻ, do đó $x^5+2023 x=x\left(x^4+2023\right) \vdots 4$ (vì $x$ lẻ nên $x \equiv 1(\bmod 4)$.
Tuy nhiên $x^5+2023 x=5^y+2 \equiv 1^y+2 \equiv 3(\bmod 4)$ (Vô lí).

Vậy không tồn tại các số nguyên $x, y$ thỏa mãn $x^5+2023 x=5^y+2$.

Ví dụ 3.31. Tìm các số nguyên $x$ và $y$ sao cho $3^x-y^3=1$.

Hướng dẫn giải

Giả sử tồn tại các số nguyên $x$ và $y$ sao cho $3^x-y^3=1$. Nhận xét $x \geq 0$.
Ta có $3^x=y^3-1=(y+1)\left(y^2-y+1\right)$, suy ra $\left\{\begin{array}{l}y+1=3^t \\\ y^2-y+1=3^{x-t}\end{array} \quad(t \in \mathbb{N}, t \leq x)\right.$.
Khi đó $y=3^t-1$ và
$$
\left(3^t-1\right)^2-\left(3^t-1\right)+1=3^{x-t} \Leftrightarrow 3^{2 t}-3^{t+1}+3=3^{x-t} .
$$

  • Nếu $t=0$, từ (3.2) ta được $1=3^x$ hay $x=0$. Ngoài ra $y=3^0-1=2$.

Nếu $t \geq 1$, giả sử $x-t \geq 2$, khi đó $3^{x-t} \vdots 9$. Từ (3.2) ta có $3^{2 t} \vdots 9$ và $3^{t+1} \vdots 9$ (do $t \geq 1$ ), từ đó suy ra $3 \vdots 9$ (Vô lí).
Do đó $x-t \in{0,1}$.

  • Nếu $x-t=0$ thì $y^2-y+1=1 \Leftrightarrow y(y-1)=0 \Leftrightarrow\left[\begin{array}{l}y=0 \ y=1\end{array}\right.$.
    Với $y=0$ ta tìm được $x=0$ và với $y=1$ ta có $3^x=2$ (Vô lí).
  • Nếu $x-t=1$ thì $y^2-y+1=3 \Leftrightarrow y^2-y-2=0 \Rightarrow y=2$.
    Khi đó $3^x=2^3+1=9$, dẫn đến $x=2$.

Vậy $(x, y)=(0,0)$ hoặc $(x, y)=(2,1)$.

Ví dụ 3.32. Tìm các số nguyên dương $x$ và $y$ sao cho
$$
9^x-7^x=2^y .
$$

Hướng dẫn giải

Giả sử tồn tại các số nguyên dương $x, y$ sao cho $9^x-7^x=2^y$.
Nếu $x$ lẻ thì
$$
9^x-7^x \equiv 1^x-(-1)^x \equiv 2(\bmod 8) .
$$

Do đó $2^y \equiv 2(\bmod 8)$, suy ra $y=1$. Khi đó $9^x-7^x=2 \Rightarrow x=1$.
Nếu $x$ chẵn, đặt $x=2 k\left(k \in \mathbb{N}^*\right)$, ta được
$$
2^y=9^{2 k}-7^{2 k}=\left(9^k-7^k\right)\left(9^k+7^k\right) .
$$

Suy ra
$$
\left\{\begin{array}{l}
9^k-7^k=2^t \\
9^k+7^k=2^{y-t}
\end{array}\right.
$$
với $t \in \mathbb{N}^*$ và $t \leq y$.
– Nếu $k$ lẻ, khi đó $2^t \equiv 9^k-7^k \equiv 2(\bmod 8)$, do đó $t=2$ và $k=1$.
Dẫn đến $x=2$ và $2^y=81-49=32 \Rightarrow y=5$.
– Nếu $k$ chẵn, ta có
$$
9^k+7^k \equiv 1^k+(-1)^k \equiv 2(\bmod 8) .
$$

Do đó $2^{y-t} \equiv 2(\bmod 8)$, suy ra $y-t=1$. Như vậy $9^k+7^k=2$ (Vồ lí).
Vậy $(x, y)=(1,1)$ hoặc $(x, y)=(2,5)$.

Ví dụ 3.33. Tìm tất cả các số nguyên tố $p$ sao cho luôn tồn tại các số nguyên dương $n, x, y$ thỏa mãn
$$
p^n=x^3+y^3 .
$$

Hướng dẫn giải

Đặt $x=p^t x_1$ và $y=p^s y_1\left(x_1, y_1, s, t \in \mathbb{N}\right.$ và $\left.x_1, y_1 \neq p\right)$.
Ta có
$$
p^n=p^{3 t} x_1^3+p^{3 s} y_1^3>p^{3 t} \Rightarrow n>3 t .
$$

Không mất tính tổng quát, giả sử $t \geq s$.
Nếu $t>s$ thì $p^{n-3 s}=p^{3(t-s)} x_1^3+y_1^3 \vdots p \Rightarrow y_1^3 \vdots p$ (Vô lí).
Vậy $t=s$, do đó $p^{n-3 t}=x_1^3+y_1^3=\left(x_1+y_1\right)\left(x_1^2-x_1 y_1+y_1^2\right)$.

  • Nếu $x_1^2-x_1 y_1+y_1^2=1$ thì $x_1=y_1=1$.
    Khi đó $p^{n-3 t}=2 \Rightarrow\left\{\begin{array}{l}p=2 \\\ n-3 t=1\end{array} \Rightarrow\left\{\begin{array}{l}p=2 \\\ n=3 t+1\end{array}\right.\right.$.
    Lúc này ta được $x=y=2^t$. Thử lại thỏa.
  • Nếu $x_1^2-x_1 y_1+y_1^2>1$, ta được
    $$
    \left\{\begin{array}{l}
    x_1+y_1=p^k \\\
    x_1^2-x_1 y_1+y_1^2=p^{n-3 t-k}
    \end{array}\right.
    $$
    với $k \geq 1, n-3 t-k \geq 1$.

Do đó $\left(x_1+y_1\right)^2-\left(x_1^2-x_1 y_1+y_1^2\right)=3 x_1 y_1 \vdots p \Rightarrow 3 \vdots p \Rightarrow p=3$.

Ngoài ra, nếu $n-3 t-k \geq 2$ thì $x_1^2-x_1 y_1+y_1^2=\left(x_1+y_1\right)^2-3 x_1 y_1 \vdots 3^2$, mà $\left(x_1+y_1\right)^2 \vdots 3^2$ nên $3 x_1 y_1 \vdots 3^2 \Rightarrow x_1 y_1 \vdots 3$ (Vô lí).
Vậy $n-3 t-k=1$ hay $x_1^2-x_1 y_1+y_1^2=3$. Không mất tính tổng quát, giả sử $x_1 \geq y_1$ thì ta được $x_1=2$ và $y_1=1$.
Từ đây ta được $n-3 t=2 \Leftrightarrow n=3 t+2$ và $x=2 \cdot 3^t$ và $y=3^t$.
Thử lại thỏa.
Vậy $p=2$ và $p=3$ là các số nguyên tố cần tìm.

Ví dụ 3.34. Tìm nghiệm tự nhiên của phương trình
$$
\left(2^x+1\right)\left(2^x+2\right)\left(2^x+3\right)\left(2^x+4\right)-5^y=11879 .
$$

Hướng dẫn giải

Giả sử tồn tại các số tự nhiên $x, y$ thỏa mãn
$$
\left(2^x+1\right)\left(2^x+2\right)\left(2^x+3\right)\left(2^x+4\right)-5^y=11879 .
$$

Ta có
$$
\begin{aligned}
\left(2^x+1\right)\left(2^x+2\right)\left(2^x+3\right)\left(2^x+4\right) & =\left(4^x+5 \cdot 2^x+4\right)\left(4^x+5 \cdot 2^x+6\right) = \left(4^x+5 \cdot 2^x+5\right)^2-1 .
\end{aligned}
$$

Do đó $\left(4^x+5 \cdot 2^x+5\right)^2-1-5^y=11879 \Leftrightarrow\left(4^x+5 \cdot 2^x+5\right)^2-5^y=11880$.
Nếu $y \geq 1$ thì ta suy ra $4^x+5 \cdot 2^x+5 \vdots 5 \Rightarrow 4^x \vdots 5$. (Vô lí)
Do đó $y=0$, khi đó
$$
\left(4^x+5 \cdot 2^x+5\right)^2=11881 \Rightarrow 4^x+5 \cdot 2^x+5=109 \Leftrightarrow 4^x+5 \cdot 2^x-104=0 .
$$

Suy ra $2^x=8 \Rightarrow x=3$.
Vậy $x=3$ và $y=0$.

Ví dụ 3.35. Cho $M=a^2+3 a+1$ với $a$ là số nguyên dương.
(a) Chứng minh rằng mọi ước của $M$ đều là số lẻ.
(b) Tìm các giá trị của $a$ để $M$ là lũy thừa của 5 .

Hướng dẫn giải

(a) Ta có $a^2+3 a+1=a(a+3)+1$ là số lẻ. Do đó mọi ước của $M$ đều là số lẻ.
(b) Giả sử tồn tại $n \in \mathbb{N}^*$ thỏa mãn $a^2+3 a+1=5^n$. Khi đó
$$
a^2+3 a-4=5^n-5 \Leftrightarrow(a+4)(a-1)=5\left(5^{n-1}-1\right) .
$$

Nếu $n>1$ thì $5^{n-1}-1>0$.
Ta lại có $(a+4)(a-1) \vdots 5$ và $a+4-(a-1)=5$ nên $\left\{\begin{array}{l}a+4 \vdots 5 \\\ a-1 \vdots 5\end{array}\right.$.
Do đó $(a+4)(a-1) \vdots 25 \Rightarrow 5\left(5^{n-1}-1\right) \vdots 25 \Rightarrow 5^{n-1}-1 \vdots 5$. (Vô lí)
Vậy $n=1$ hay $a^2+3 a+1=5 \Rightarrow a=1$.
Thử lại thỏa, vậy $M$ là lũy thừa của 5 khi và chỉ khi $a=1$.

Ví dụ 3.36. Tìm số tự nhiên $n$ sao cho $8^n+47$ là số nguyên tố.

Hướng dẫn giải

  • Xét $n=2 k(k \in \mathbb{N})$, khi đó
    $$
    p^n \equiv 8^n+47 \equiv(-1)^{2 k}+47 \equiv 48 \equiv 0(\bmod 3) .
    $$

Do đó $p$ ! 3 nên $p$ không là số nguyên tố (Vô lí).

  • Xét $n=4 k+1\left(k \in \mathbb{N}^*\right)$, khi đó
    $$
    p \equiv\left(8^4\right)^k \cdot 8+47 \equiv 8+47 \equiv 55 \equiv 0(\bmod 5) .
    $$

Do đó $p \vdots: 5$ nên $p$ không là số nguyên tố (Vô lí).

  • Nếu $n=4 k+3\left(k \in \mathbb{N}^*\right)$, khi đó
    $$
    p \equiv\left(8^4\right)^k \cdot 8^3+47 \equiv 8^3+47 \equiv 559 \equiv 0(\bmod 13) .
    $$

Do đó $p$ : 13 nên $p$ không là số nguyên tố (Vô lí).
Vậy không tồn tại số tự nhiên $n$ để $8^n+47$ là số nguyên tố.

Ví dụ 3.37. Cho phương trình $2^x+5^y=k^2$ ( $x, y, k$ là các số nguyên dương).
(a) Chứng minh rằng phương trình trên vô nghiệm khi $y$ chẵn.
(b) Tìm $k$ để phương trình có nghiệm.
(Đề thi tuyển sinh vào lớp 10 chuyên toán PTNK 2022)

Hướng dẫn giải

(a) Giả sử tồn tại $y \in \mathbb{N}^*$ chẵn để phương trình trên có nghiệm.

  • Với $x=1$ thì $2+5^y=k^2 \equiv 2(\bmod 5)$.
    Điều này vô lý vì $k^2 \equiv 0,1,4(\bmod 5)$ với mọi $k \in \mathbb{N}$.
  • Với $x>1$, do $y$ chẵn nên ta đặt $y=2 m(m \in \mathbb{N})$.
    Khi đó
    $$
    2^x+5^{2 m}=k^2 \Leftrightarrow 2^x=\left(k-5^m\right)\left(k+5^m\right) \Rightarrow\left\{\begin{array}{l}
    k-5^m=2^t \\\
    k+5^m=2^{x-t}
    \end{array} \quad(t \geq 0) .\right.
    $$

Vì $k+5^m>k-5^m$ nên $x-t>t$, suy ra $k=2^{t-1}+2^{x-t-1}$.
Ta thấy nếu $t=0$ thì $k=\dfrac{1}{2}+2^{x-1} \notin \mathbb{N}$. Do đó $t \geq 1$.

Mặt khác $k$ lẻ và $t-1<x-t-1$ nên $2^{t-1}=1 \Rightarrow t=1$. Khi đó $k-5^m=2 \Leftrightarrow k=2+5^m$. Thay vào $2^x+5^{2 m}=k^2$, ta được
$$
2^x+5^{2 m}=\left(2+5^m\right)^2 \Leftrightarrow 2^x=4+2 \cdot 5^m .
$$

Vì $x>1$ nên $2^x \vdots 4$, suy ra $2 \cdot 5^m \vdots 4$ (Vô lí).
Vậy phương trình vô nghiệm khi $y$ chẵn.
(b) Giả sử phương trình có nghiệm, khi đó $y$ lẻ.

  • Nếu $x=4 z+1(z \in \mathbb{N})$ thì
    $$
    k^2 \equiv 2^x+5^y \equiv 2^{4 z} \cdot 2+5^y \equiv 2(\bmod 5) .
    $$

Điều này vô lý vì $k^2 \equiv 0,1,4(\bmod 5)$ với mọi $k \in \mathbb{N}$.

  • Nếu $x=4 z+3(z \in \mathbb{N})$ thì
    $$
    k^2 \equiv 2^{4 z} \cdot 2^3+5^y \equiv 8 \equiv 3(\bmod 5) \text { (Vô lí). }
    $$

Vậy $x$ chẵn, đặt $x=2 t\left(t \in \mathbb{N}^*\right)$.
Ta có
$$
2^x+5^y=k^2 \Leftrightarrow 5^y=\left(k-2^t\right)\left(k+2^t\right) \Rightarrow\left\{\begin{array}{l}
k-2^t=5^s \\\
k+2^t=5^{y-s}
\end{array} \quad(s \in \mathbb{N}) .\right.
$$

Nếu $s>0$ thì $5^{y-s}-5^s \vdots 5$ nên $2^{t+1} \vdots 5$ (vô lý). Do đó $s=0$.

Khi đó $\left\{\begin{array}{l}k=1+2^t \\\ k=5^y-2^t\end{array}\right.$. Suy ra $1+2^t=5^y-2^t \Rightarrow 5^y-1=2^{t+1}$.
Nếu $t>1$ thì $2^{t+1} \vdots 8$. Dặt $y=2 l+1$, khi đó
$$
2^{t+1}=5^y-1=25^l \cdot 5-1 \equiv 5-1 \equiv 4(\bmod 8) \text{vô lý}
$$

Vậy $t=1$, suy ra $k=3$. Với $k=3$, ta tìm được $x=2$ và $y=1$.
Vậy phương trình có nghiệm khi và chỉ khi $k=3$.

Ví dụ 3.38. Cho $k$ là số nguyên dương và $a=3 k^2+3 k+1$.
(a) Chứng minh rằng $2 a$ và $a^2$ là tổng của ba số chính phương.
(b) Chứng minh rằng nếu $a$ là uớc của số nguyên $b$ và $b$ bằng tổng của ba số chính phương thì bất kì lũy thừa với số mũ nguyên dương nào của $b$ cũng là tổng của ba số chính phương.

Hướng dẫn giải

(a) Ta có
$$
\begin{aligned}
2 a=6 k^2+6 k+2 & =k^2+\left(k^2+2 k+1\right)+\left(4 k^2+4 k+1\right) = k^2+(k+1)^2+(2 k+1)^2
\end{aligned}
$$
$$
\begin{aligned}
a^2 & =\left(3 k^2+3 k-1+2\right)^2=9 k^4+18 k^3+15 k^2+6 k+1 = \left(4 k^4+12 k^3+13 k^2+6 k+1\right)+\left(4 k^4+4 k^3+k^2\right)+\left(k^4+2 k^3+k^2\right) = \left(2 k^2+3 k+1\right)^2+\left(2 k^2+k\right)^2+\left(k^2+k\right)^2
\end{aligned}
$$
(b) Đặt $a^2=a_1^3+a_2^3+a_3^3$ với $a_1, a_2, a_3 \in \mathbb{Z}$.
Đặt $b=c a$ với $c$ là số nguyên dương, do $b$ bẳng tổng của ba số chính phương nên $b=b_1^2+b_2^2+b_3^2$ với $b_1, b_2, b_3$ là các số nguyên.
Xét số nguyên dương $n$ bất kì, khi đó

  • Nếu $n=2 k\left(k \in \mathbb{Z}^{+}\right)$thì
    $$
    \begin{aligned}
    b^n & =c^{2 k} a^{2 k}=\left(c^k a^{k-1}\right)^2 a^2 = \left(c^k a^{k-1}\right)^2\left(a_1^2+a_2^2+a_3^2\right) = \left(c^k a^{k-1} a_1\right)^2+\left(c^k a^{k-1} a_2\right)^2+\left(c^k a^{k-1} a_3\right)^2
    \end{aligned}
    $$
  • Nếu $n=2 k+1(k \in \mathbb{Z})$ thì
    $$
    b^n=\left(b^k\right)^2 \cdot b=\left(b^k\right)^2\left(b_1^2+b_2^2+b_3^2\right)=\left(b^k b_1\right)^2+\left(b^k b_2\right)^2+\left(b^k b_3\right)^2
    $$

Hoàn tất chứng minh.


C. CÁC BÀI TẬP RÈN LUYỆN

Bài 3.13. Tìm nghiệm nguyên dương của phương trình
$$
x^3+x^2+x+1=2011^y .
$$

Bài 3.14. Tìm tập nghiệm nguyên dương của phương trình
$$
8^x+15^y=17^z .
$$

Bài 3.15. Tìm các số nguyên dương $x, y, z>1$ thỏa mãn
$$
(x+1)^y-x^z=1 .
$$

Bài 3.16. Tìm nghiệm tự nhiên của phương trình $5^x-3^y=2$.

Bài 3.17. Tìm nghiệm nguyên dương của phương trình
$$
2^x \cdot 3^y+5^z=7^t .
$$

Bài 3.18. Cho các số nguyên dương $m, n \geq 2$. Tìm nghiệm nguyên dương của phương trình
$$
x^n+y^n=3^m .
$$

Bài 3.19. Cho $p$ là một số nguyên tố và $a, n$ là các số nguyên dương. Chứng minh rằng nếu $2^p+3^p=$ $a^n$ thì $n=1$.

Bài 3.20. Chứng minh rằng tích của ba số nguyên liên tiếp không thể là lũy thừa với số mũ lớn hơn 1 của một số nguyên.

Bài 3.21. Cho phương trình $3 x^2-y^2=23^n$ với $n$ là số tự nhiên.
(a) Chứng minh nếu $n$ chẵn thì phương trình đã cho không có nghiệm nguyên $(x, y)$.
(b) Chứng minh nếu $n$ lẻ thì phương trình đã cho có nghiệm nguyên $(x, y)$.

Bài 3.22.
(a) Cho $m$ là số nguyên. Chứng minh rằng nếu tồn tại các số nguyên $a, b, c$ khác 0 sao cho $a+b+c=0$ và $a b+b c+c a+4 m=0$ thì cũng tồn tại các số nguyên $a^{\prime}, b^{\prime}, c^{\prime}$ sao cho $a^{\prime}+b^{\prime}+c^{\prime}=0$ và $a^{\prime} b^{\prime}+b^{\prime} c^{\prime}+a^{\prime} c^{\prime}+m=0$.
(b) Với $k$ là số nguyên dương, chứng minh rằng không tồn tại các số nguyên $a, b, c$ khác 0 sao cho $a+b+c=0$ và $a b+b c+c a+2^k=0$.
(Đề thi tuyển sinh lớp 10 chuyên Toán PTNK 2015)

ƯỚC CHUNG VÀ MỘT SỐ ÁP DỤNG

Bài viết của thầy Nguyễn Vĩnh Khang – Giáo viên Star Education

Các tính chất của ước chung

Nhận xét: Nếu ta đặt $(x, y)=d$, thì $x^{\prime}=\dfrac{x}{d}$ và $y^{\prime}=\dfrac{y}{d}$ nguyên tố cùng nhau. Từ đó lợi dụng các tính chất liên quan đến số nguyên tố cùng nhau như (được sử dụng thẳng, không cần chứng minh)

  • Nếu $a b: c$, và $(b, c)=1$, ta có $a: c$.
  • Nếu $a: b$ và $a: c$, với $(b, c)=1$, ta có $a: b c$.
  • Nếu $(a, b)=1, r$ là ước của $a, s$ là ước của $b$, ta cũng có $(r, s)=1$.
    để phân tích bài toán tiếp. Việc đặt ước chung như vậy sẽ làm đơn giản bài toán (do ta có thể rút $d$ ra rồi triệt tiêu, nếu được) và cho thêm dữ kiện $\left(x^{\prime}, y^{\prime}\right)=1$.

Tính chất 3.1. Giả sử $a, b, c, n$ là các số nguyên dương, chứng minh những tính chất sau
(a) $\operatorname{gcd}(a, b, c)=\operatorname{gcd}(\operatorname{gcd}(a, b), c)$
(b) $\operatorname{gcd}(a c, b c)=\operatorname{gcd}(a, b) c$
(c) Nếu $\operatorname{gcd}(a, b)=1$, ta có $\operatorname{gcd}(a b, c)=\operatorname{gcd}(a, c) \operatorname{gcd}(b, c)$
(d) $\operatorname{gcd}\left(a^n, b^n\right)=\operatorname{gcd}(a, b)^n$.

Chứng minh.
Phần 1: gọi $d=\operatorname{gcd}(a, b, c)$ ta có $d$ là ước của $a, b$, nên $\operatorname{gcd}(a, b)$ : $d$. Nhưng $c: d$, nên ta được một chiều
$$
\operatorname{gcd}(\operatorname{gcd}(a, b), c) \vdots d=\operatorname{gcd}(a, b, c)
$$

Để chứng minh chiều còn lại, gọi $d=\operatorname{gcd}(\operatorname{gcd}(a, b), c)$. Tương tự như trên ta có $d$ là ước của $\operatorname{gcd}(a, b)$, nên $d$ cũng là ước của $a, b$. Nhưng $d$ là ước của $a, b, c$, nên
$$
\operatorname{gcd}(a, b, c) \vdots d=\operatorname{gcd}(\operatorname{gcd}(a, b), c)
$$

Kết hợp (1.1) và (1.2), ta có đpcm.

Phần 2: nếu $d=(a c, b c)$, ta có $d: c$ do $c$ là ước chung của $a c, b c$. Đặt $d=k c$, ta có $(a c, b c)=k c$, và $a c, b c: k c$. Nói cách khác $a, b: k$, nên $(a, b): k$, và
$$
c(a, b) \vdots k c=(a c, b c)
$$

Mặt khác, đặt $k=(a, b)$, ta có $a, b: k$, nên $a c, b c: k c$. Theo định nghīa, $(a c, b c) \vdots k c=(a, b) c$. Kết hợp với (2.1) ta có đpem $\operatorname{gcd}(a c, b c)=\operatorname{gcd}(a, b) c$.

Phần 3: gọi $k=\operatorname{gcd}(a, c), l=\operatorname{gcd}(b, c)$, theo tính chất 2 , ta được
$$
\left\{\begin{array}{l}
\operatorname{gcd}\left(\dfrac{a}{k}, \dfrac{c}{k}\right)=1 \\
\operatorname{gcd}\left(\dfrac{b}{l}, \dfrac{c}{l}\right)=1
\end{array}\right.
$$

Mặt khác $a: k, b: l$, nhưng $a, b$ lại nguyên tố cùng nhau, nên $k, l$ cūng vậy. Kết hợp với $c: k, l$, ta có $c: k, l$. Để ý rằng $\dfrac{c}{k l}$ là ước của $\dfrac{c}{k}$ và $\dfrac{c}{l}$, nên
$$
\left\{\begin{array}{l}
\operatorname{gcd}\left(\dfrac{a}{k}, \dfrac{c}{k l}\right)=1 \\
\operatorname{gcd}\left(\dfrac{b}{l}, \dfrac{c}{k l}\right)=1
\end{array}\right.
$$

Ta chứng minh $\operatorname{gcd}(a b, c)=1$ nếu $\operatorname{gcd}(a, b)=\operatorname{gcd}(b, c)=\operatorname{gcd}(a, c)=1$. Thật vậy, giả sử ngược lại, tức $\operatorname{gcd}(a b, c) \neq 1$. Khi đó tồn tại $p$ là ước nguyên tố chung của $a b, c$. Nhưng $a b: p$ thì ta phải có $a: p$ hoặc $b: p$, nên $\operatorname{gcd}(a, c): p$ hoặc $\operatorname{gcd}(b, c)$ : $($ cả 2 đều mâu thuẫn với giả thiết).

Áp dụng quan sát trên cho (3.1), ta được
$$
\operatorname{gcd}\left(\dfrac{a b}{k k}, \dfrac{c}{k l}\right)=1 \Leftrightarrow \operatorname{gcd}(a b, c)=k l=\operatorname{gcd}(a, c) \operatorname{gcd}(b, c)
$$

Phần 4: ta chứng minh $\operatorname{gcd}\left(a^n, b^n\right)=1$ nếu $\operatorname{gcd}(a, b)=1$. Thật vậy, giả sử $\operatorname{gcd}\left(a^n, b^n\right) \neq 1$, khi đó $a^n, b^n$ phải có một ước nguyên tố chung $p$. Sử dụng tính chất nếu $x y: p$ thì $x: p$ hoặc $y: p$. Từ đó $a, b: p$, vô lý.

Đặt $d=\operatorname{gcd}(a, b)$, ta có $\operatorname{gcd}\left(\dfrac{a}{d}, \dfrac{b}{d}\right)=1$, nên
$$
\operatorname{gcd}\left(\left(\dfrac{a}{d}\right)^n,\left(\dfrac{b}{d}\right)^n\right)=1
$$

Nhân $d^n$ cho cả 2 vế, và dùng tính chất 2 , ta được
$$
\operatorname{gcd}(a, b)^n=d^n=d^n \operatorname{gcd}\left(\left(\dfrac{a}{d}\right)^n,\left(\dfrac{b}{d}\right)^n\right)=\operatorname{gcd}\left(a^n, b^n\right)
$$

Hệ quả 3.1
Giả sử $a, b, c, n$ là các số nguyên dương, chứng minh những tính chất sau
(a) Nếu $a b: c$ và $(a, b)=1$, tồn tại $k, l$ sao cho $k l=c$, và $a: k, b \vdots l$.
(b) Nếu $a b=c^n$ và $(a, b)=1(n \geq 2)$, tồn tại $k, l$ sao cho $k l=c$ và $a=k^n, b=l^n$.

Chứng minh.

Phần 1: gọi $k=\operatorname{gcd}(a, c), l=\operatorname{gcd}(b, c)$, theo bài tập trước, ta có $k l=\operatorname{gcd}(a, c) \operatorname{gcd}(b, c)=$ $\operatorname{gcd}(a b, c)=c$, và $a: k, b: l$ theo định nghĩa.

Phần 2: gọi $k=\operatorname{gcd}(a, c), l=\operatorname{gcd}(b, c)$, theo bài tập trước, ta có $k l=\operatorname{gcd}(a, c) \operatorname{gcd}(b, c)=\operatorname{gcd}(a b, c)=$ c. Mặt khác
$$
k^n=\operatorname{gcd}\left(a^n, c^n\right)=\operatorname{gcd}\left(a^n, a b\right)=a \operatorname{gcd}\left(a^{n-1}, b\right)=a
$$
, ở đây $\operatorname{gcd}\left(a^{n-1}, b\right)=1$ do nếu tồn tại $p$ là ước nguyên tố chung cho $a^{n-1}, b$, ta phải có $p$ là ước chung của $a, b$ (vô lý). Chứng minh tương tự, ta cũng có $l^n=b$. Ta có đpcm.

B. MỘT SỐ VÍ DỤ ÁP DỤNG
Ví dụ 3.1 (Junior Balkan Mathematical Olympiad 2001).
Tìm ước chung lớn nhất của $A_0, A_1, A_2, \ldots, A_{1999}$, với $A_n=2^{3 n}+3^{6 n+2}+5^{6 n+2}$.

Hướng dẫn giải

Do $A_0=35=5 \cdot 7$, nên ước chung lớn nhất, gọi là $d$, phải là 1 trong 4 số ${1,5,7,35}$. Do $A_1=$ $2^3+3^8+5^8 \equiv 8+(-2)^8 \equiv 4(\bmod 5)$ nên $d \neq 5,35$. Mặt khác, theo định lý Fermat, ta có $3^6 \equiv 5^6$ $(\bmod 7)$, nên
$$
A_n \equiv 8^n+\left(3^6\right)^n \cdot 9+\left(5^6\right)^n \cdot 25 \equiv 1+9+25 \equiv 0 \quad(\bmod 7)
$$
Ta kết luận $d=7$.


Ví dụ 3.2. Chứng minh rằng nếu $d>0$ không phải là số chính phương, thì $\sqrt{d}$ là số vô tỷ.

Hướng dẫn giải

Để ý rằng $d=1^2 \cdot d$ nên $d$ luôn có thể viết thành dạng $d=x^2 y$ (với $x, y>0$ ). Chọn $x$ lớn nhất có thể, và để ý $y \neq 1$. Nếu $y$ có ước chính phương $z^2$ ngoài 1 , thì $d=x^{\prime 2} y^{\prime}$, với $x^{\prime}=x z>x$ và $y^{\prime}=\dfrac{y}{z}$, vô lý. Như vậy $y$ là tích các số nguyên tố khác nhau (do nếu $p$ là ước nguyên tố của $y$, thì $\dfrac{y}{p}$ không thể nào chia hết cho $p$ được).

Giả sử $\sqrt{d}=\dfrac{a}{b}$ là một số hữu tỷ, với $a, b$ nguyên dương nguyên tố cùng nhau. Ta có $$ a^2=b^2 d=(b x)^2 \cdot y $$ nên $a^2: y$. Nhưng $y$ chỉ là tích các số nguyên tố khác nhau, nên $a: y$. Thế $a=c y$ vào (*), ta được
$$
c^2 y^2=(b x)^2 y \Leftrightarrow b^2 x^2=c^2 y
$$

Để ý $c^2 y: b^2$, nhưng $(c, b)=1$ (do $(a, b)=1$ ), nên $y: b^2$. Ta đã chọn sao cho $y$ không thể nào có ước chính phương nào ngoài 1 , nên $b=1$ ! Từ đó ta có $\sqrt{d}=a$, hay $d=a^2$, vô lý.


Hướng dẫn giải

Gọi $d>0$ là một ước chung của $a^m+b^n, a^m-b^n$. Khi đó $\left\{\begin{array}{I}2 a^m=\left(a^m+b^n\right)+\left(a^m-b^n\right) \vdots d \\\ 2 b^n=\left(a^m+b^n\right)-\left(a^m-b^n\right) \vdots d\end{array}\right.$.
Để ý rằng $a, b$ khác tính chẵn lẻ, nên $a^m+b^n$ và $a^m-b^n$ luôn lẻ. Nhưng $d$ là một ước chung, nên $d$ lẻ. Như vậy $a^m, b^n: d$.

Nếu $d \neq 1$, gọi $p$ là một ước nguyên tố của $d$ (có thể $d=p$ ). Khi đó $a^m, b^n: p$, nên ta cũng có $a, b: p$. Điều này mâu thuẫn với giả thiết $a, b$ nguyên tố cùng nhau, nên $d=1$. Nhưng $d$ bất kỳ, nên $a^m+b^n, a^m-b^n$ chỉ có ước chung (dương) là 1 . Hay nói cách khác, $a^m+b^n, a^m-b^n$ nguyên tố cùng nhau.

Ví dụ 3.4. Cho 2 số hữu tỷ $\dfrac{a}{b}, \dfrac{c}{d}$ viết ở dạng tối giản (tức $(a, b)=(c, d)=1$ ) sao cho $d\frac{a}{b}+\dfrac{c}{d}$ là một số nguyên. Chứng minh rằng $|b|=|d|$.

Hướng dẫn giải

Ta có $\dfrac{a}{b}+\dfrac{c}{d}=\dfrac{a d+b c}{b d}$ là một số nguyên, nên $a d+b c: b$, hay $a d: b$. Nhưng $a, b$ nguyên tố cùng nhau, nên $d: b$.
Chứng minh tương tự với $a d+b c: d$, ta có $b: d$. Như vậy $|b|=|d|$.

Ví dụ 3.5 (Spanish Mathematical Olympiad 1996).
Giả sử $a, b$ là các số nguyên dương sao cho $\dfrac{a+1}{b}+\dfrac{b+1}{a}$ là số nguyên. Nếu $d$ là ước chung lớn nhất của $a, b$
(a) Chứng minh rằng $a+b \geq d^2$.
(b) Tìm một cặp $(a, b)$ mà $a+b=d^2$.

Hướng dẫn giải

(a) Đặt $a=d a^{\prime}, b=d b^{\prime}$, ta có
$$
\dfrac{a+1}{b}+\dfrac{b+1}{a}=\dfrac{d^2\left(a^{\prime 2}+b^{\prime 2}\right)+d\left(a^{\prime}+b^{\prime}\right)}{d^2 a^{\prime} b^{\prime}} \in \mathbb{Z}
$$
nên $\dfrac{d^2\left(a^{\prime 2}+b^{\prime 2}\right)+d\left(a^{\prime}+b^{\prime}\right)}{d^2}=a^{\prime 2}+b^{\prime 2}+\dfrac{a^{\prime}+b^{\prime}}{d}$ cūng là số nguyên. Như vậy $a^{\prime}+b^{\prime}: d$. Nhưng $a, b$ nguyên dương, nên $a^{\prime}+b^{\prime} \geq d$, hay $a+b=d\left(a^{\prime}+b^{\prime}\right) \geq d^2$.
(b) $a=3, b=6$, thì $\dfrac{a+1}{b}+\dfrac{b+1}{a}=3$ và $a+b=9=\operatorname{gcd}(a, b)^2$.

Ví dụ 3.6 (Romanian Mathematical Olympiad 2003).
Cho $n$ là một số chẵn nguyên dương. Tìm tất cả các số nguyên dương $a, b$ sao cho $a^n+b^n: a+b$.

Hướng dẫn giải

Do $n$ chẵn ta có $a^n-b^n: a^2-b^2: a+b$. Như vậy
$$
\left\{\begin{array}{l}
2 a^n=\left(a^n+b^n\right)+\left(a^n-b^n\right) \vdots a+b \\\
2 b^n=\left(a^n+b^n\right)-\left(a^n-b^n\right) \vdots a+b
\end{array}\right.
$$

Gọi $d=(a, b)$, và $a=d u, b=d v$, ta có $u, v$ nguyên tố cùng nhau và $\operatorname{gcd}(a, b)=2 d^n \operatorname{gcd}\left(u^n, v^n\right)=$ $2 d^n: d(u+v)$. Nói cách khác, $2 d^{n-1}: u+v$.

Để cho ra tất cả giá trị $a, b$ có thể, ta bắt đầu với 2 số $u, v$ nguyên dương và nguyên tố cùng nhau. Tiếp theo chọn $d$ bất kỳ sao cho $2 d^{n-1}: u+v(d$ luôn tồn tại do ta có thể chọn $d=u+v)$. Khi đó $a=d u, b=d v$ thỏa mãn đề bài.

Thật vậy, từ $a^n+b^n=d^n\left(u^n+v^n\right)$, ta chia làm 2 trường hợp
(a) Nếu $u, v$ đều lẻ: ta có $u^n+v^n$ chẵn, nên $a^n+b^n: 2 d^n: d(u+v)=a+b$.
(b) Nếu, không mất tính tổng quát, $u$ chẵn, $v$ lẻ: do $2 d^{n-1}: u+v$, và $u+v$ lẻ, nên $d^{n-1}: u+v$. Từ đó $a^n+b^n: d^n: d(u+v)=a+b$.

Ta kết luận $a=d u, b=d v$, với $u, v$ nguyên tố cùng nhau sao cho $u+v$ là ước của $2 d^{n-1}$.

Ví dụ 3.7 (India Mathematical Olympiad 1998).
Tìm tất cả các bộ số nguyên dương $(x, y, n)$ sao cho
$$
\operatorname{gcd}(x, n+1)=1 \text { và } x^n+1=y^{n+1} .
$$

Hướng dẫn giải

Do $x>0$, nên $y^{n+1}=x^n+1>1$. Ta có
$$
x^n=y^{n+1}-1=(y-1)\left(y^n+y^{n-1}+\cdots+y+1\right)
$$

Do $y-1>1$, ta phải có $y-1: p$ với $p$ là một ước nguyên tố nào đó của $x$. Từ đó
$$
y^n+y^{n-1}+\cdots+y+1 \equiv \underbrace{1+1+\cdots+1}_{n \text { số } 1} \equiv n+1 \quad(\bmod p)
$$

Như vậy $p$ là ước chung của $x$ và $n+1$, vô lý.

Ví dụ 3.8 (Bulgarian Mathematical Olympiad 2001).

Tìm tất cả các bộ $(a, b, c)$ nguyên dương sao cho $a^3+b^3+c^3$ chia hết cho $a^2 b, b^2 c$, và $c^2 a$.

Hướng dẫn giải

Đầu tiên để ý rằng nếu $d$ là ước chung của $a, b$, ta có $a^3+b^3+c^3: a^2 b: d^3$, nên $c: d$. Như vậy nếu ta đặt $d=(a, b, c)$, và $a=d u, b=d v, c=d w, u, v$ phải nguyên tố cùng nhau. Chứng minh tương tự, ta có $u, v, w$ đôi một nguyên tố cùng nhau.

Do $a^3+b^3+c^3: a^2 b$, ta có
$$
d^3\left(u^3+v^3+w^3\right): d^3 u^2 v \Leftrightarrow u^3+v^3+w^3: u^2 v
$$

Từ đó, $u^3+v^3+w^3: u^2$, và $v^3+w^3: u^2$. Chứng minh tương tự, ta cūng có $u^3+v^3+w^3: v^2, w^2$, và $w^3+u^3: v^2, u^3+v^3: w^2$. Nhưng $u, v, w$ nguyên tố cùng nhau đôi một, nên
$$
\left\{\begin{array}{l}
u^3+v^3+w^3: u^2 v^2 w^2 \\\
v^3+w^3: u^2 \\\
w^3+u^3: v^2 \\\
u^3+v^3: w^2
\end{array}\right.
$$

Không mất tính tổng quát, giả sử $u \leq v \leq w$. Do $a, b, c$ nguyên dương, $u, v, w$ cũng nguyên dương, và $u^2 v^2 w^2 \leq u^3+v^3+w^3 \leq 3 w^3$. Nói cách khác, $w \geq \dfrac{u^2 v^2}{3}$. Mặt khác, $u^3+v^3: w^2$, nên ta được
$$
u^3+v^3 \geq w^2 \geq \dfrac{u^4 v^4}{9} (*)
$$

Nhưng $u \leq v$, nên $2 v^3 \geq u^3+v^3 \geq \frac{u^4 v^4}{9}$, hay $u^4 v \leq 18$. Ta suy ra $u=1$ hoặc $u=2$. Nhưng $u=2$ thì $v \geq 2$, nên $32 \leq u^4 v \leq 18$, vô lý.
*Như vậy $u=1$. Nếu $v=1$ thì 2 : $w^2$, cho nên $w=1$. Ta có bộ $(a, b, c)=(d, d, d)$ thỏa mãn. Nếu $v \geq 2$, ta phải có $w>v$, hay $w \geq v+1 \geq 3$ do $v, w$ nguyên tố cùng nhau. Nhưng $u^3+v^3+w^3: u^2 v^2 w^2$, nên ta có
$$
1+v^3+w^3: v^2 w^2 \Rightarrow v^2 w^2 \leq 1+v^3+w^3 \leq 1+(w-1)^3+w^3<2 w^3
$$

Chia $w^2$ cho cả 2 vế, ta được $v^2<2 w$, hay $w>\frac{v^2}{2}$. Mặt khác, ta có $v^3+u^3: w^2$, nên
$$
v^3+1 \geq w^2>\frac{v^4}{4} \Leftrightarrow 4>v^3(v-4)
$$

Vậy $v \leq 4$. Nhưng $v \geq 2$, ta xét các trường hợp sau
(a) $v=4$ : khi đó $u^3+v^3=65: w^2$, nên $w=1$ (vô lý do $v \leq w$ ).
(b) $v=3$ : khi đó $u^3+v^3=28: w^2$, nên $w \in{1,2}$ (cũng vô lý như trên).
(c) $v=2$ : khi đó $u^3+v^3=9: w^2$, nên $w=3$ (do $w \geq v$ ).

Kiểm tra lại, ta nhận $(a, b, c)=(d, 2 d, 3 d)$ và các hoán vị của nó. Ta kết luận
$$
\begin{aligned}
& (a, b, c)=(k, k, k),(k, 2 k, 3 k),(k, 3 k, 2 k), \
& \quad(2 k, k, 3 k),(2 k, 3 k, k),(3 k, k, 2 k),(3 k, 2 k, k) \quad k \geq 1
\end{aligned}
$$

Ví dụ 3.9. Cho các số nguyên dương $x, y, z$ sao cho $\dfrac{1}{x}+\dfrac{1}{y}=\dfrac{1}{z}$. Giả sử $x, y, z$ nguyên tố cùng nhau (tức $(x, y, z)=1$ ), chứng minh rằng $x+y$ là một số chính phương.

Hướng dẫn giải

Viết lại phương trình thành $x+y=\dfrac{x y}{z}$. Đặt $d=(x, y)$, và $x=d a, y=d b$, ta có $(d, z)=1$ (do $(x, y, z)=1)$ và $(a, b)=1$. Thêm nữa
$$
a+b=\dfrac{d a b}{z}
$$

Ta có $d a b: z$, và $(d, z)=1$, nên $a b: z$. Do $(a, b)=1$, ta sẽ chứng minh $z$ có thể tách thành $z=r s$ sao cho $a: r$ và $b: s$.
*Đặt $r=(a, z)$ và $s=(b, z)$.

  • Theo định nghĩa $a: r, b: s$. Nhưng $(a, b)=1$, nên $(r, s)=1$.
  • Tương tự z:r,s. Kết hợp với điều chứng minh ở trên, ta có $z$ :rs.
  • Mặt khác, đặt $a=k r, b=l s$ và $z=q(r s)$, ta có $k l r s: q r s$, nên $k l: q$. Ta cũng có $(a, z)=r$, nên $(k, q s)=1$.

Như vậy $k l: q$ và $(k, q)=1$. Chứng minh tương tự, ta có $(l, q)=1$. Từ đó $q=1$, và $z=r s$.
Tóm tắt lại, ta có $a=k r, b=l s$ và $z=r s$.
*Thế vào $a+b=\dfrac{d a b}{z}$, ta có
$$
k r+l s=d k l
$$

Để ý $(a, b)=1$ nên $(k, l s)=1$. Mặt khác, $l s=d k l-k r: k$, cho nên $k=1$. Chứng minh tương tự, ta có $l=1$, nên $a b=r s=z$, và $a+b=\dfrac{d a b}{z}=d$. Từ đó $x+y=d(a+b)=d^2$ là một số chính phương.

Ví dụ 3.10. Giải phương trình nghiệm nguyên sau (theo các biến $x, y, n, m$ ) với $m, n \geq 0$.
$$
x^n+y^n=2^m
$$

Hướng dẫn giải

Đặt $d=(x, y)>0$ và $x=d u, y=d v$, ta có $u, v$ nguyên tố cùng nhau và $d^n\left(u^n+v^n\right)=2^m$. Như vậy $d=2^e\left(0 \leq e \leq \frac{m}{n}\right)$. Đặt $k=2^{m-n e}$, ta xét phương trình sau (với $u, v$ nguyên tố cùng nhau).
$$
u^n+v^n=2^k
$$
(a) Nếu $n$ chẵn

(a) Nếu $n=0$ : phương trình gốc trở thành $2^m=2$, nên $m=1$. Ta nhận bộ nghiệm $(x, y, 0,1)$ với mọi $x, y \neq 0$.

(b) Nếu $n \geq 2$ :
i. Nếu $k=0$ : ta có $u^n+v^n=1$. Nhưng $n$ chẵn, nên phương trình chỉ có 4 nghiệm $(0, \pm 1)$ và $( \pm 1,0)$. Ta nhận bộ
$$
(x, y, m, n)=\left( \pm 2^e, 0, n e, n\right),\left(0, \pm 2^e, n e, n\right) \quad(n \text { chẵn })
$$
ii. Nếu $k \geq 1$ : ta có $u^n+v^n$ chẵn. Kết hợp với $u, v$ nguyên tố cùng nhau, ta được $u, v$ cùng lẻ. Xét modulo 4, ta có $2^k=u^n+v^n \equiv 1+1 \equiv 2(\bmod 4)$. Nói cách khác $k=1$ và $u^n+v^n=2$, hay $u, v= \pm 1$. Ta nhận bộ
$$
(x, y, m, n)=\left( \pm 2^e, \pm 2^e, n e+1, n\right) \quad(n \text { chẵn })
$$

(b) Nếu $n$ lẻ: ta có $2^k=(u+v)\left(u^{n-1}-u^{n-2} v+\cdots+v^{n-1}\right)$, nên $u+v=2^s$ với $s \geq 0$.
(a) Nếu $n=1$ : ta có $u+v=2^k$, nên ta nhận các bộ sau
$$
(x, y, m, n)=\left(u, 2^m-u, m, 1\right) \quad(u \text { nguyên bất kỳ })
$$

(b) Nếu $n \geq 3$ :
i. Nếu $k=0$ : ta có $u^n+v^n=1$. Nhưng $u^n+v^n=(u+v)\left(u^{n-1}-u^{n-2} v+\cdots-u v^{n-2}+v^{n-1}\right)$, cho nên $u+v= \pm 1$.
*Với $v=1-u$, ta xét phương trình sau
$$
u^n-(u-1)^n=1
$$

Ta có $u=0,1$ là nghiệm, cho nên ta nhận các bộ sau
$$
(x, y, m, n)=\left(2^e, 0, n e, n\right),\left(0,2^e, n e, n\right) \quad(n \text { lẻ })
$$

Nếu $u \geq 2$, ta chứng minh
$$
u^n-(u-1)^n>1
$$
với mọi $n \geq 2$ bằng quy nạp. Khi $n=2$, ta có $u^2-(u-1)^2=2 u-1 \geq 3>1$. Giả sử bất đẳng thức đúng với $n$, ta chứng minh nó đúng với $n+1$
$$
\begin{aligned}
u^{n+1}-(u-1)^{n+1} & =u^n+(u-1) u^n-(u-1)^{n+1} \
& =u^n+(u-1)\left[u^n-(u-1)^n\right] \
& \geq 2^n+(2-1) \cdot 1>1
\end{aligned}
$$

Nếu $u \leq-1$, ta cũng chứng minh $u^n-(u-1)^n=(1-u)^n-(-u)^n>1$ với mọi $n \geq 2$ bằng quy nạp. Khi $n=2$, ta có $(1-u)^n-(-u)^n=-2 u+1>1$. Giả sử bất đẳng thức đúng với $n$, ta chứng minh nó đúng với $n+1$
$$
\begin{aligned}
(1-u)^{n+1}-(-u)^{n+1} & =(1+w)^{n+1}-w^{n+1} \quad(\text { đặt } w=-u \geq 1) \
& =w(w+1)^n+(w+1)^n-w^{n+1} \
& =(w+1)^n+w\left[(w+1)^n-w^n\right] \
& \geq 2^n+1 \cdot 1>1
\end{aligned}
$$
*Với $v=-1-u$, ta xét phương trình sau
$$
u^n-(u+1)^n=1 \Leftrightarrow(u+1)^n-u^n=-1
$$

Dùng những bất đẳng thức ta đã chứng minh ở trên, cộng với trường hợp $u=0,1$ không thỏa, ta kết luận trường hợp này vô nghiệm.
ii. Nếu $k \geq 1$ : ta có $u^n+v^n$ chẵn, và $u, v$ nguyên tố cùng nhau, nên $u, v$ cùng lẻ. Như vậy
$$
u^{n-1}-u^{n-2} v+\cdots-v^{n-1} \equiv \underbrace{1+1+\cdots+1}_{n \text { số } 1} \equiv n \equiv 1 \quad(\bmod 2)
$$

Kết hợp với $(u+v)\left(u^{n-1}-u^{n-2} v+\cdots+v^{n-1}\right)=2^k$, ta phải có

Kết hợp với $(u+v)\left(u^{n-1}-u^{n-2} v+\cdots+v^{n-1}\right)=2^k$, ta phải có
$$
\left\{\begin{array}{l}
u+v=2^k \quad(k \geq 1) \\\
u^{n-1}-u^{n-2} v+\cdots+v^{n-1}=1
\end{array}\right.
$$

Để ý $u^n+v^n=2^k=u+v$, ta sẽ chứng minh

$\left(u^n-v^n\right)(u-v) \geq 0$ với mọi $u, v$ và $n$ lẻ bằng quy nạp lên $n$. Trường hợp $n=1$ chính là $(u-v)^2 \geq 0$, còn $n=3$ là $\left(u^3-v^3\right)(u-v)=(u-v)^2\left(u^2+u v+v^2\right) \geq 0$.

Giả sử nó đúng với $n-2$ và $n$, ta chứng minh nó cũng đúng với $n+2$
$$
\begin{aligned}
\left(u^{n+2}-v^{n+2}\right)(u-v)= & \left(u^{n+2}-u^2 v^n+u^2 v^n-u^n v^2+u^n v^2-v^{n+2}\right)(u-v) \
= & u^2\left(u^n-v^n\right)(u-v)+u^2 v^2\left(u^{n-2}-v^{n-2}\right)(u-v) \
& +v^2\left(u^n-v^n\right)(u-v) \geq 0
\end{aligned}
$$

  • $2\left(u^n+v^n\right) \geq\left(u^2+v^2\right)\left(u^{n-2}+v^{n-2}\right)$ với mọi $u, v$ và $n \geq 3$ lẻ. Thật vậy, bất đẳng thức tương đương với
    $$
    u^n+v^n-u^2 v^{n-2}-v^2 u^{n-2} \geq 0 \Leftrightarrow\left(u^{n-2}-v^{n-2}\right)(u-v) \geq 0
    $$

Đúng theo bất đẳng thức ta đã chứng minh ở trên.
Áp dụng vào bài toán, ta có $u+v=u^n+v^n \geq \frac{u^2+v^2}{2} \cdot\left(u^{n-2}+v^{n-2}\right) \geq\left(\frac{u^2+v^2}{2}\right)^2$. $\left(u^{n-4}+v^{n-4}\right) \geq \cdots\left(\frac{u^2+v^2}{2}\right)^{(n-1) / 2}(u+v)$. Nhưng $u+v=2^k \geq 2^1>1$, nên
$$
\left(\frac{u^2+v^2}{2}\right)^{(n-1) / 2} \leq 1 \Leftrightarrow u^2+v^2 \leq 2
$$

Xét các giá trị $u, v=0, \pm 1$ thỏa mãn điều kiện trên, ta được các cặp $(u, v)=$ $(0,0),( \pm 1,0),(0, \pm 1),( \pm 1, \pm 1)$. Thử vào $u+v=u^n+v^n=2^k$ (với $2^k \geq 2^1=2$ ), ta chỉ có đúng $u=v=1$ và $k=1$ thỏa. Ta nhận các bộ
$$
(x, y, m, n)=\left(2^e, 2^e, n e+1, n\right) \quad(n \geq 3 \text { lẻ })
$$

Tổng hợp các trường hợp lại, ta kết luận các nghiệm $(x, y, m, n)$ như sau
(a) $\left(2^e, 0, n e, n\right),\left(0,2^e, n e, n\right)$, và $\left(2^e, 2^e, n e+1, n\right)(e, n \geq 0)$.
(b) $\left(-2^e, 0, n e, n\right),\left(0,-2^e, n e, n\right)$, và $\left( \pm 2^e, \pm 2^e, n e+1, n\right)(e, n \geq 0, n$ chẵ $)$.
(c) $\left(u, 2^m-u, m, 1\right)(u \in \mathbb{Z}, m \geq 0)$

Ví dụ 3.11. Tìm tất cả các số nguyên dương $x, y, z$ sao cho
$$
16 x y z=d(x+y+z)^2
$$
với $d$ là ước chung của $x, y, z$

Hướng dẫn giải

Đặt $x=d a, y=d b, z=d c$, ta có $(a, b, c)=1$. Phương trình tương đương với $16 a b c=(a+b+c)^2$.
*Gọi $p^{2 k+1}$ là một ước của $a, p$ nguyên tố. Ta sẽ chứng minh $p^{2 k+2}$ cũng là ước của $a$.

(a) Nếu $p=2$ : đặt $a=2^{2 k+1} u$, ta có
$$
2^{2 k+5} u b c=\left(2^{2 k+1} u+b+c\right)^2
$$

Nếu $b$ chẵn thì $c$ cũng phải chẵn (và ngược lại), nhưng điều này mâu thuẫn với $a, b, c$ nguyên tố cùng nhau. Như vậy $b, c$ phải lẻ. Đê ý vế trái là bội của $2^{2 k+5}$ (mũ lẻ), nên $Q^2: 2^{2 k+5}(Q=$ $2^{2 k+1} u+b+c$. Nói cách khác, $Q: 2^{k+3}$ hay $Q=2^{k+3} R$. Từ đó
$$
2^{2 k+5} u b c=Q^2=2^{2 k+6} R^2
$$
nên $2^{2 k+5} u b c: 2^{2 k+6}$. Nhưng $b, c$ lẻ, nên ta có $u: 2$. Như vậy $a=2^{2 k+1} u: 2^{2 k+2}$.
(b) Nếu $p>2$ : lập luận tương tự như trên, ta đặt $Q=a+b+c$ và $a=p^{2 k+1} u$. Phương trình tương đương với
$$
Q^2=16 p^{2 k+1} u b c: p^{2 k+1}
$$
hay $Q: p^{k+2}$. Ta có $16 p^{2 k+1} u b c=Q^2: p^{2 k+2}$. Nhưng $p>2$, nên $u b c: p$.

Giả sử, không mất tính tổng quát b:p. Khi đó $(a+b+c)^2=16 a b c: p$, nên $a+b+c: p$. Nhưng $a: p$, nên c:p. Ta có điều vô lý do $a, b, c$ nguyên tố cùng nhau. Như vậy $u: p$, nên $a=p^{2 k+1} u: p^{2 k+2}$.

Như vậy nếu $a=p_1^{\alpha_1} p_2^{\alpha_2} \cdots p_m^{\alpha_m}$ là phân tích thừa số nguyên tố, các số mũ $\alpha_i$ phải chẵn (nếu $\alpha_i$ lẻ thì $p_i^{\alpha_i+1}$ cũng là ước của $a$, vô lý). Cùng với $a>0$, ta kết luận $a$ là số chính phương. Chứng minh tương tự, $b, c$ cũng chính phương.
*Đặt tiếp $a=u^2, b=v^2, c=w^2(u, v, w>0)$, ta có phương trình
$$
16 u^2 v^2 w^2=\left(u^2+v^2+w^2\right)^2 \Leftrightarrow u^2+v^2+w^2=4 u v w
$$

Do $a, b, c$ nguyên tố cùng nhau, $u, v, w$ cũng phải nguyên tố cùng nhau. Mặt khác, xét modulo 4 cho cả 2 vế, ta có $u^2+v^2+w^2 \equiv 0,1,2,3(\bmod 4)$, với $u^2+v^2+w^2 \equiv 0(\bmod 4)$ khi và chỉ khi $u^2, v^2, w^2 \equiv 0 (\text{b mod 4} )$. Như vậy $u, v, w$ đều chẵn, vô lý.

Ta kết luận phương trình vô nghiệm.

Bài toán hàm số trong kì thi tuyển sinh vào 10

Trong các kì thi tuyển sinh vào 10 có dạng toán liên quan đến hàm số, chủ yếu là hàm bậc hai dạng $y = ax^2$ (1) và đường thẳng $y = mx + n$ (2)Trong bài viết này chủ yếu xét các bài toán tương giao giữa đồ thị hàm số (1) và (2).

Nếu hàm số $y =ax^2$ có đồ thị là parabol $(P)$ và hàm số $y = mx + n$ có đồ thị là đường thẳng $d$, thì phương trình hoành độ giao điểm của $(P)$ và $(d)$ là

$$ax^2 = mx + n \Leftrightarrow ax^2 – m x – n =0 (*)$$

$(*)$ là một phương trình bậc hai, nên có 3 trường hợp xảy ra:

  • TH1: Nếu $(*)$ vô nghiệm thì $(d)$ và $(P)$ không có giao điểm.
  • TH2: Nếu $(*)$ có 1 nghiệm thì $(d)$ và $(P)$ có 1 giao điểm, ta nói $d$ tiếp xúc với $(P)$.
  • TH3: Nếu $(*)$ có hai nghiệm phân biệt thì ta nói $(d)$ cắt $(P)$, và nghiệm của $(*)$ là hoành độ của hai giao điểm, từ hoành độ ta có thể tính tung độ của giao điểm dựa vào phương trình của $(d)$ hoặc của $(P)$.

Ta xét một vài ví dụ sau:

Bài 1. (Thi vào lớp 10 trường PTNK năm 2018) Gọi $(P),(d)$ lần lượt là đồ thị của các hàm số $y=x^2$ và $y=2 m x+3$.
a) Chứng minh đường thẳng $(d)$ luôn cắt $(P)$ tại hai điểm phân biệt $A\left(x_1 ; y_1\right), B\left(x_2 ; y_2\right)$ và tính $y_1+y_2$ theo $m$.
b) Tìm $m$ sao cho $y_1-4 y_2=x_1-4 x_2+3 x_1 x_2$.

Lời giải bài 1.
a) Phương trình hoành độ giao điểm của $(P)$ và $(d)$ là:
$$
x^2=2 m x+3 \Leftrightarrow x^2-2 m x-3=0 \quad(1)
$$

Xét phương trình (1), ta có: $\Delta^{\prime}=m^2+3>0$ với mọi $m \in \mathbb{R}$
Suy ra phương trình (1) luôn có hai nghiệm phân biệt $x_1, x_2$ với mọi $m$ hay $(d)$ luôn cắt $(P)$ tại hai điểm phân biệt $A\left(x_1 ; y_1\right), B\left(x_2 ; y_2\right)$.
Theo định lý Viete, ta có: $\left\{\begin{array}{l}x_1+x_2=2 m \\\ x_1 x_2=-3\end{array}\right.$
Khi đó $y_1=2 m x_1+3, y_2=2 m x_2+3$
$y_1+y_2=2 m x_1+3+2 m x_2+3=2 m\left(x_1+x_2\right)+6=4 m^2+6$
b) Ta có:
$y_1-4 y_2=x_1-4 x_2+3 x_1 x_2 $
$\Leftrightarrow 2 m x_1+3-8 m x_2-12=x_1-4 x_2-9 $
$ \Leftrightarrow 2 m\left(x_1-4 x_2\right)=x_1-4 x_2 $
$ \Leftrightarrow\left(x_1-4 x_2\right)(2 m-1)=0 $
$ \Leftrightarrow\left[\begin{array}{l}
x_1=4 x_2 \\\
m=\frac{1}{2} \quad(n)
\end{array}\right. $
Với $x_1=4 x_2 $ lại có $x_1 x_2=-3 \Rightarrow 4 x_2^2=-3 $ (vô lý)
Vậy $m=\frac{1}{2} $

Bài 2. (Đề thi vào 10 trường PTNK năm 2019) Cho $(P),(d)$ lần lượt là đồ thị hàm số $y=x^2$ và $y=2 x+m$.
a) Tìm $m$ sao cho $(P)$ cắt $(d)$ tại hai điểm phân biệt $A\left(x_1 ; y_1\right), B\left(x_2 ; y_2\right)$.
b) Tìm $m$ sao cho $\left(x_1-x_2\right)^2+\left(y_1-y_2\right)^2=5$.

Lời giải bài 2.
a) Phương trình hoành độ giao điểm của $(P)$ và $(d)$
$$
x^2=2 x+m \Leftrightarrow x^2-2 x-m=0 \quad(1)
$$
$(P)$ cắt $(d)$ tại 2 điểm phân biệt $A, B \Leftrightarrow (1)$ có 2 nghiệm phân biệt

$\Leftrightarrow \Delta^{\prime}>0 \Leftrightarrow 1+m>0 $
$ \Leftrightarrow m>-1(*)$
Vậy $m>-1$ thì $(P)$ cắt $(d)$ tại hai điểm phân biệt.


b) Với điều kiện $(*)$ theo Viete ta có: $S=x_1+x_2=2, P=x_1 \cdot x_2=-m$

Ta có: $A\left(x_1 ; y_1\right) \in(d) \Leftrightarrow y_1=2 x_1+m ; B\left(x_2 ; y_2\right) \in(d) \Leftrightarrow y_2=2 x_2+m$

Ta có: $\left(x_1-x_2\right)^2+\left(y_1-y_2\right)^2=5 $

$\Leftrightarrow\left(x_1-x_2\right)^2+\left(2 x_1-2 x_2\right)^2=5$

$\Leftrightarrow\left(x_1-x_2\right)^2+4\left(x_1-x_2\right)^2=5 $

$\Leftrightarrow\left(x_1-x_2\right)^2=1 \Leftrightarrow\left(x_1+x_2\right)^2-4 x_1 x_2=1$

$\Leftrightarrow 4+4 m=1 \Leftrightarrow m=\frac{-3}{4} $ thỏa (*)
Vậy $m = \dfrac{-3}{4}$.

Bài 3. Đồ thị của hàm số $f(x)=a x^2$ và $g(x)=-a x+b(a ; b$ là các số thực), điểm chung thứ nhất có hoành độ bằng 1 và tung độ điểm chung thứ 2 là 8 . Tìm hoành độ của điểm chung thứ hai của hai đồ thị và tính $a, b$.

Lời giải bài 3.

  • Phương trình hoành độ giao điểm $a x^2=-a x+b \Leftrightarrow a x^2+a x-b=0$ thì phương trình nhận 1 là nghiệm nên $a 1^2+a \cdot 1-b=0 \Rightarrow b=2 a$.
  • Khi đó gọi nghiệm còn lại là $x_2$ ta có $1 \cdot x_2=\frac{-b}{a}=-2$
  • Do đó tung độ $a(-2)^2=8$, suy ra $a=2$ và $b=4$.

Bài 4. (TS chuyên Đăk Lăk 2020 – 2021) Trong mặt phẳng $O x y$, cho parabol $(P): y=x^2$ và đường thẳng $(d): y=2(m+1) x+3$ với $m$ là tham số. Tìm tất cả các giá trị của tham số $m$ để đường thẳng $(d)$ cắt parabol tại hai điểm phân biệt có hoành độ $x_1, x_2$ thoả mãn điều kiện $x_1^2-2 m x_1+2 x_2-x_1 x_2=2$.

Lời giải bài 4.

  • Phương trình hoành độ giao điểm $x^2-2(m+1) x-3=0\left(^*\right)$ $\Delta^{\prime}=(m+1)^2+3>0$ với mọi $m$.
  • Theo định lý Viete ta có $x_1+x_2=2(m+1), x_1 x_2=-3$.
    Ta có $x_1^2-2(m+1) x_1-3=0$, suy ra $x_1^2-2 m x_1=2 x_1+3$ $x_1^2-2 m x_1+2 x_2-x_1 x_2=2 \Leftrightarrow 2 x_1+3+2 x_2-(-3)=2 \Leftrightarrow m=-2$.
  • Vậy $m=-2$.

Bài 5. (TS chuyên Khánh Hoà 2020 – 2021) Trên mặt phẳng toạ độ $O x y$, cho parabol $(P)$ có phương trình $y=2 x^2$ và đường thẳng $(d): y=-2 m x+m+1$ với $m$ là tham số.
a) Chứng minh đường thẳng $(d)$ luôn cắt Parabol $(P)$ tại hai điểm phân biệt.
b) Gọi $x_1, x_2$ lần lượt là hoành độ giao điểm của đường thẳng $(d)$ và parabol $(P)$, tìm $m$ thoả mãn đẳng thức $\frac{1}{\left(2 x_1-1\right)^2}+\frac{1}{\left(2 x_2-1\right)^2}=66$.

Lời giải bài 5 .
a) Phương trình hoành độ giao điểm của $d$ và $P$ là
$$
2 x^2+2 m x-m-1=0
$$
$\Delta^{\prime}=m^2-2(-m-1)=(m+1)^2+1>0$ với mọi $m$, do đó $d$ cắt $P$ tại hai điểm phân biệt với mọi $m$.
b) Theo định lý Viete ta có $x_1+x_2=-m, x_1 x_2=\frac{-m-1}{2}$.
Suy ra $x_1^2+x_2^2=\left(x_1+x_2\right)^2-2 x_1 x_2=m^2+m+1$
Ta có $66=\frac{1}{\left(2 x_1-1\right)^2}+\frac{1}{\left(2 x_2-1\right)^2}=\frac{\left(2 x_1-1\right)^2+\left(2 x_2-1\right)^2}{\left(2 x_1-1\right)^2\left(2 x_2-1\right)^2}=\frac{4\left(x_1^2+x_2^2\right)-4\left(x_1+x_2\right)+2}{\left(4 x_1 x_2-2\left(x_1+x_2\right)+1\right)^2}$
$$
=\frac{4\left(m^2+m+1\right)-4(-m)+2}{(-2 m-2-2(-m)+1)^2}=\frac{4 m^2+8 m+6}{1}
$$

Giải ra được $m=-5, m=3$.

Bài 6. (TS chuyên Thái Bình 2020 – 2021) Trong mặt phẳng toạ độ $O x y$, cho parabol $(P): y=\frac{x^2}{2}$ và hai đường thẳng $\left(d_1\right): y=5 x+2,\left(d_2\right): y=\left(m^2+1\right) x+m$ với $m$ là tham số.
a) Tìm $m$ để $\left(d_1\right)$ song song với $\left(d_2\right)$.
b) Tìm $m$ để $\left(d_2\right)$ cắt parabol $(P)$ tại hai điểm phân biệt có hoành độ $x_1, x_2$ sao cho $Q=x_1+x_2-4 x_1 x_2$ đạt giá trị nhỏ nhất.

Lời giải bài 6 .
a) Điều kiện để $d_1 || d_2$ là $m^2+1=5, m \neq 2$, giải ra được $m=-2$.
b) Phương trình hoành độ giao điểm của $d_2$ và $P$ là
$$
\frac{x^2}{2}=\left(m^2+1\right) x+m \Leftrightarrow x^2-2\left(m^2+1\right) x-2 m=0
$$

Điều kiện $\Delta^{\prime}=\left(m^2+1\right)^2-(-2 m)>0 \Leftrightarrow m^4+2 m^2+1+2 m>0 \Leftrightarrow m^4+m^2+(m+1)^2>0$ (Đúng với mọi $m)$

Theo định lý Viete ta có $x_1+x_2=2\left(m^2+1\right), x_1 x_2=-2 m$

Ta có $P=x_1+x_2-4 x_1 x_2=$ $2\left(m^2+1\right)-4(-2 m)=2\left(m^2+1+4 m\right)=2(m+2)^2-6 \geq-6$, đẳng thức xảy ra khi $m=-2$.

Bài 8. Trong mặt phẳng tọa độ $O x y$, cho parabol $(P): y=x^2$ và đường thẳng $(d): y=2 x-m-2$. Tìm tất cả các giá trị của tham số $m$ để $(d)$ cắt $(P)$ tại hai điểm phân biệt lần lượt có hoành độ $x_1, x_2$ thỏa mãn $x_1^2+1=2 x_2$.

Lời giải bài 8 .

  • Phương trình hoành độ giao điểm
    $$
    x^2=2 x-m-2 \Leftrightarrow x^2-2 x+m+2=0
    $$
  • Điều kiện $\Delta^{\prime}=1-(m+2)>0 \Leftrightarrow m<-1$.
  • Theo định lý Viete ta có $x_1+x_2=2, x_1 x_2=m+2$.

Ta có $x_1^2=2 x_1-m-2$, suy ra $x_1^2+1=2 x_2 \Leftrightarrow 2 x_1-m-2+1=2 x^2 \Leftrightarrow 2\left(x_1-x_2\right)=m+1$ Kết hợp với Viete ta có $x_1=\frac{m+5}{4}, x_2=\frac{3-m}{4}$
Khi đó $x_1 x_2=m+2 \Leftrightarrow \frac{m+5}{4} \frac{3-m}{4}=m+2 \Leftrightarrow m=-1(l), m=-17(n)$.

  • Vậy $m=-17$.

Bài 9. Cho $(P): y=x^2$ và đường thẳng $(d): y=(m+2) x-2 m$.
a) Tìm $m$ để $d$ cắt $(P)$ tại hai điểm phân biệt $A\left(x_1 ; y_1\right), B\left(x_2 ; y_2\right)$.
b) Tìm $m$ để $x_1+2 y_2=7$.

Lời giải bài 9 .
a) Phương trình hoành độ giao điểm

$\quad x^2-(m+2) x+2 m=0 $
$\Delta=(m+2)^2-8 m=(m-2)^2>0 \Leftrightarrow m \neq 2 .$

b) Khi đó phương trình có nghiệm $x=2, x=m$.
3

  • TH1: $x_1=2, x_2=m$ suy ra $y_1=4, y_2=m^2$. Ta có $2+2 m^2=7$ giải ra được $m=\sqrt{2,5}, m=$ $-\sqrt{2,5}$.
  • TH2: $x_1=m, x_2=2$, suy ra $y_1=m^2, y_2=4$. Ta có $m+2.4=7 \Leftrightarrow m=-1$.
  • Vậy có 3 giá trị $m$ thỏa đề bài $m=\sqrt{2,5}, m=-\sqrt{2,5}, m=-1$.

Bài 10. Trong mặt phẳng tọa độ $O x y$, cho parabol $(P)$ có phương trình $y=x^2$ và đường thẳng $(d)$ có phương trình $y=2 m x-m^2-m-2$ (với $m$ là tham số).
a) Tìm tọa độ điểm $M$ thuộc $(P)$ biết điểm $M$ có hoành độ bằng -3 .
b) Tìm điều kiện của $m$ để đường thẳng $(d)$ cắt parabol $(P)$ tại hai điểm phân biệt. Gọi $A\left(x_1 ; y_1\right), B\left(x_2 ; y_2\right)$ là hai giao điểm của đường thẳng $(d)$ và parabol $(P)$, xác định $m$ để $x_1 y_2+x_2 y_1=2 m^3+6$.

Lời giải bài 10.

b) Tìm điều kiện của $m$ để đường thẳng $(d)$ cắt parabol $(P)$ tại hai đie biệt. Gọi $A\left(x_1 ; y_1\right), B\left(x_2 ; y_2\right)$ là hai giao điểm của đường thẳng $(d)$ và $(P)$, xác định $m$ để $x_1 y_2+x_2 y_1=2 m^3+6$. Ta có phương trình hoành độ giao điểm của $(d)$ và $(P)$ là

$ x^2=2 m x-m^2-m-2 \Leftrightarrow x^2-2 m x+m^2+m+2=0(1) $
$ \Delta^{\prime}=(-m)^2-\left(m^2+m+2\right)=-m-2$

$(d)$ cắt parabol $(P)$ tại 2 điểm phân biệt khi và chỉ khi phương trình (1) có hai nghiệm phân biệt $\Leftrightarrow \Delta^{\prime}>$ $0 \Leftrightarrow-m-2>0 \Leftrightarrow m<-2(*)$

$ \text { Ta có } x_1+x_2=2 m, x_1 x_2=m^2+m+2 $
$x_1 y_2+x_2 y_1=x_1 \cdot x_2^2+x_2 \cdot x_1^2=x_1 \cdot x_2\left(x_1+x_2\right)=2$ $m\left(m^2+m+2\right) $
$=2 m^3+2 m^2+4 m $
$2 m^3+2 m^2+4 m=2 m^3+6 \Leftrightarrow 2 m^2+4 m-6=0 $

$\Leftrightarrow\left[\begin{array}{l}
m=1 \\\
m=-3
\end{array}\right.$

Đối chiếu (*) vậy $m=-3$.

Một số bài toán về đường cao và trực tâm

Trong chương trình hình học chuyên toán dành cho lớp 9, có nhiều bài toán liên quan đến các đường cao và trực tâm tam giác, hôm nay chúng ta sẽ tìm hiểu một số tính chất và bài tập như thế.

Bài 1. Cho tam giác $ABC$ nội tiếp đường tròn $(O)$, các đường cao $AD, BE, CF$ cắt nhau tại $H$. $M$ là trung điểm của $BC$. Vẽ đường kính $AK$.

a) Chứng minh $H, M, K$ thẳng hàng;

b) Chứng minh $AH = 2 \cdot OM$.

c) Gọi $G$ là trọng tâm tam giác $ABC$. Chứng minh $H, G, O$ thẳng hàng và $GH = 2 \cdot OG$.

Hướng dẫn - Gợi ý

(a) Ta có $\angle A B K=\angle A C K=90^{\circ}$, từ đó $B K / / C H, C K / / B H$, do đó tứ giác $B H C K$ là hình bình hành.
Hơn nữa $M$ là trung điểm $B C$ nên cũng là trung điểm $H K$.
(b) Tam giác $A H K$ thì $O, M$ lần lượt là trung điểm $A K, H K$ nên $O M$ là đường trung bình, do đó $A H=2 \cdot O M$.
(c) Tam giác $A B C$ có $A M$ là trung tuyến nên với $G$ là trọng tâm thì ta có $A G=\frac{2}{3} A M$. Hơn nữa tam giác $A H K$ có $A M$ là trung tuyến và $G$ thuộc $A M$ và $A G=\frac{2}{3} A M$ nên $G$ cũng là trọng tâm tam giác $A H K$; Mặt khác có $O$ là trung điểm của $A K$ nên $G$ thuộc đoạn $H O$ và $G H=2 \cdot G O$.

Bài 2. Cho tam giác $A B C$ nhọn nội tiếp đường tròn $(O)$. Các đường cao $A D, B E, C F$ cắt nhau tại $H$. $A O$ căt $E F$ tại $K$ và $(O)$ tại $L$.
a) Chứng minh $\angle B A H=\angle C A O$ và $\angle A O \perp E F$.
b) $C F, B E$ cắt $(O)$ tại $Q, P$. Chứng minh $A P=A Q=A H$.
c) Tính $\angle A$ nếu $B, H, O, C$ cùng thuộc một đường tròn. Khi đó tính $\angle O H C$.

Hướng dẫn - Gợi ý

(a) Ta có $\angle B A D=90^{\circ}-\angle A B D, \angle C A K=90^{\circ}-\angle A K C$;
Hơn nữa $\angle A B D=\angle A K C$, suy ra $\angle B A D=\angle C A K$.
Gọi $L$ là giao điểm của $A K$ và $E F$.
Tứ giác $B F E C$ nội tiếp, suy ra $\angle A E F=\angle A B C=\angle A K C$, do đó tứ giác $E L K C$ nội tiếp.
Từ đó $\angle A L E=\angle A C K=90^{\circ}$ hay $A O \perp E F$.

(b) Ta có $\angle A P H=\angle A C B=\angle A H P $ suy ra $ A H=A P$. Tương tự thì $A H=A Q$

(c) Ta có $\angle B H C=\angle E H F=180^{\circ}-\angle B A C$; Và $\angle B O C=2 \angle B A C$; Khi đó $B H O C$ nội tiếp khi và chỉ khi $\angle B H C=\angle B O C$ hay $180^{\circ}-\angle B A C=2 \angle B A C$, tính ra $\angle B A C=60^{\circ}$.
Khi đó $\angle O H C=\angle O B C=30^{\circ}$.

Bài 3. Cho tam giác $A B C$, các đường cao $A D, B E, C F$ cắt nhau tại trực tâm tam giác là $H$. Gọi $M$ là trung điểm $B C$ và $P$ là hình chiếu vuông góc của $H$ trên $A M$. Chứng minh rằng
(a) Các tứ giác $B F P M, C E P M$ nội tiếp.
(b) Tứ giác $B H P C$ nội tiếp.
(c) $B C$ là tiếp tuyến chung của đường tròn ngoại tiếp tam giác $A P B, A P C$.

Hướng dẫn - Gợi ý

(a) Ta chứng minh được
$$
A P \cdot A M=A H \cdot A D=A F \cdot A B=A E \cdot A C
$$
suy ra $B F P M, C E P M$ là các tứ giác nội tiếp.
(b) Ta có $\angle B H C=180^{\circ}-\angle B A C$ và
$$
\angle B P C=\angle B P M+\angle C P M=\angle B F M+\angle C E M=\angle A B C+\angle A C B=180^{\circ}-\angle B A C
$$

Suy ra $\angle B H C=\angle B P C$ nên $B H P C$ nội tiếp.
(c) Ta có
$$
\angle M P B=\angle A B M \Rightarrow \angle M B P=\angle M A B
$$
do đó $B M$ là tiếp tuyến của đường tròn ngoại tiếp tam giác $A B P$. Tương tự thì $M C$ cũng là tiếp tuyến của đường tròn ngoại tiếp tam giác $A P C$.

Bài 4. Cho tam giác $A B C$ nhọn nội tiếp đường tròn $(O)$. Các đường cao $B E, C F$ cắt nhau tại $H$. Gọi $M$ là trung điểm $A H$.
a) Chứng minh $M E D F$ nội tiếp.
b) $M E, M F$ là tiếp tuyến của đường tròn đường kính $B C$.
c) Gọi $K$ là giao điểm $A D$ và $E F ; T$ là giao điểm của $M B$ và đường tròn đường kính $B C$. Chứng minh rằng $T, K, C$ thẳng hàng và $K$ là trực tâm tam giác $M B C$.

Hướng dẫn - Gợi ý

(a) Gọi $I$ là trung điểm $B C$, thì $I$ là tâm đường tròn đường kính $B C$.
Ta có $\angle A E H=\angle A F H=90^{\circ}$ nên $A E H F$ nội tiếp và $M$ là tâm đường tròn.
Tam giác $M A E$ và $I C E$ cân nên $\angle M E A=\angle M A E, \angle I E C=\angle I C E$, suy ra $\angle M E A+\angle I E C=$ $\angle M A E+\angle I C E=90^{\circ}$, do đó $\angle M E I=90^{\circ}$, hay $M E$ là tiếp tuyến của đường tròn $(I)$ đường kính $B C$.
Tương tự thì $M F$ cũng là tiếp tuyến của đường tron2h $(I)$ đường kính $B C$.
(b) Ta có $\angle I E M=\angle I M F=\angle I D M=90^{\circ}$, do đó 5 điểm $I, M, D, E, F$ cùng thuộc đường tròn đường kính $I M$. Do đó tứ giác $M E D F$ nội tiếp.
(c) Gọi $L$ là giao điểm $I M$ và $E F$ thì ta có $I M \perp E F$ tại $L$. Khi đó ta có $M L \cdot M I=M E^2$ (1) Hơn nữa $\triangle M E T \backsim \triangle M B E$, suy ra $M T \cdot M B=M E^2$ Và $\triangle M K L \backsim \triangle M D I$, suy ra $M K \cdot M D=M L \cdot M I$ Từ (1), (2), (3) ta có $M T \cdot M B=M K \cdot M D$, từ đó ta có $B T K D$ nội tiếp, suy ra $\angle B T K=
180^{\circ}-\angle B D K=90^{\circ} \text {. }
$

Mặt khác $\angle B K C=90^{\circ}$, từ đó $T, K, C$ thẳng hàng.
Tam giác $M B C$ có hai đường cao $M D$ và $C T$ cắt nhau tại $K$ nên $K$ là trực tâm của tam giác.

Bài 5. Cho tam giác $A B C$ nhọn nội tiếp đường tròn $(O)$. Các đường cao $B E, C F$ cắt nhau tại $H$. Gọi $M$ là trung điểm $B C$.Đường tròn đường kính $A H$ cắt $(O)$ tại $P$ khác $A$. $A P$ cắt $B C$ tại $K$.
a) Chứng minh các tứ giác $K B F P, K C E P$ nội tiếp.
b) Chứng minh $K, E, F$ thẳng hàng.
c) Chứng minh $H$ là trực tam giác $A K M$.

Hướng dẫn - Gợi ý

(a) Ta có các tứ giác $A P F E, B F E C$ nội tiếp, suy ra $\angle K P F=\angle A E F, \angle A E F=\angle C B F$, từ đó $\angle K P F=\angle C B F$, kéo theo tứ giác $K B F P$ nội tiếp. Tương tự thì $\angle A P E=\angle A F E=\angle A C K$, do đó $K P E C$ nội tiếp.
(b) Tứ giác $\angle K P F B, K P E C$ nội tiếp, suy ra $\angle P K F=\angle P B F, \angle P K E=\angle A C P$ mà $\angle P B F=$ $\angle A C P$ nên $\angle P K F=\angle P K E$, từ đó ta có $K, F, E$ thẳng hàng.
(c) Ta có $A P F E$ nội tiếp và $A F H E$ nội tiếp nên $A, P, F, H, E$ cùng thuộc một đường tròn, suy ra $\angle A P H=\angle A F H=90^{\circ}$;
Vẽ đường kính $A D$, ta có $\angle A P D=90^{\circ}$; Do đó $P, H, D$ thẳng hàng.
Mặt khác, theo bài tập 1 thì $H, M, D$ thẳng hàng nên $P, H, M$ thẳng hàng và $M P \perp A K$. Tam giác $A K M$ có $A H, P M$ là hai đường cao cắt nhau tại $H$ nên $H$ là trực tâm tam giác, do đó $K H \perp A M$.

Bài 6. Cho tam giác $A B C$, các đường cao $B E, C F$. Các điểm $P \in B E, Q \in C F$ sao cho $\angle P A B=$ $\angle Q A C=90^{\circ}$. Chứng minh rằng đường thẳng qua $A$ vuông góc $P Q$ đi qua trung điểm $B C$.

Hướng dẫn - Gợi ý

Bài 7. Cho tam giác $A B C$ nội tiếp đường tròn tâm $O$ có trực tâm $H$. Đường trung trực $A H$ cắt $A B, A C$ tại $Q, P$. Chứng minh $O A$ là phân giác $\angle P O Q$.

Hướng dẫn - Gợi ý
  • $\angle B A H=\angle O A C, \angle B A O=\angle C A H$;
  • Ta có thể suy nghĩ hướng chứng minh $\triangle A O Q \backsim \triangle A H B$.

Theo nhận xét trên, thì ta cần chứng minh $A O \cdot A H=A B \cdot A Q$, thực vậy ta có hai tam giác $Q A H$ và $O A B$ cân tại $Q, O$ và $\angle Q A H=\angle O A B$ nên
$$
\triangle Q A H \backsim \triangle O A B \Rightarrow A O \cdot A H=A Q \cdot A B
$$

Từ đó ta có $\triangle A O P \backsim \triangle A B H$, kéo theo $\triangle A O Q=\angle A B H$
Chứng minh tương tự thì $\angle A O P=\angle A C H$
Từ (1) và (2) ta có $\angle A O Q=\angle A O P$.

Bài 8. Cho tam giác $A B C$ nội tiếp đường tròn $(O)$, các đường cao $B D, B E, C F$ cắt nhau tại $H$. $A D$ cắt $(O)$ tại $K . K F$ cắt $(O)$ tại $P$.
a) Chứng minh $F H \cdot F C=F P \cdot F K$.
b) $C P$ cắt $D E$ tại $L$. Chứng minh $H L P F$ nội tiếp.
c) Chứng minh $C P$ qua trung điểm của $E F$.

Hướng dẫn - Gợi ý

(a) Ta có tứ giác $A B D E$ nội tiếp nên $\angle C E D=\angle A B C$ mà $\angle A B C=\angle A P C$, suy ra $\angle A P C=$ $\angle C E D$, từ đó có tứ giác $A P L E$ nội tiếp. Khi đó
$$
C L \cdot A P=C E \cdot C A=C H \cdot C F
$$
suy ra $H L P F$ nội tiếp.
(b) $H L$ cắt $A C$ tại $T$. Ta cần chứng minh $H T / / F E$ và $L$ là trung điểm $H T$. Ý đầu tiên ta có thể làm như sau: tứ giác $H F P L, A F H E, A P K C$ nội tiếp nên
$$
\angle C H L=\angle C P F=\angle C A K=\angle C F E
$$
suy ra $H T / / F E$.
Tiếp theo
$$
\angle T H E=\angle F E H=\angle D E H
$$

$$
\angle H E T=90^{\circ} \Rightarrow \angle L E T=\angle L T E
$$
do đó
$$
L H=L E=L T
$$

Khi đó
$$
\frac{H L}{F M}=\frac{C L}{C M}=\frac{L T}{M E} \Rightarrow M F=M E
$$

Bài 9. Cho tam giác $A B C$ có các đường cao $B D, C E$ cắt nhau tại $H$. Gọi $K$ là hình chiếu vuông góc của $H$ trên $D E . M$ là trung điểm của $B C, L$ là giao điểm của $A M$ và $D E$. Chứng minh 4 điểm $B, C, L, K$ cùng thuộc một đường tròn.

Hướng dẫn - Gợi ý

Nhận xét: Đây là bài toán khó đối với các em THCS, tuy vậy dựa vào các bài toán đã giải ở trên ta có thể kết nối để cho ra lời giải bài toán này.
Ở đây việc kéo dài $K L$ cắt $B C$, có lẽ là ý nghĩa tự nhiên nhất để sử dụng tam giác đồng dạng, suy ra tứ giác nội tiếp.
Gọi giao điểm đó là $T$, thì ta cần chứng minh $T K \cdot T L=T B \cdot T C$, hơn nữa từ bài toán 1.4 ta có tính chất $T H \perp A M$, đây có lẽ là chìa khóa để cho ta lời giải bài toán này.

Do có sử dụng bài toán phụ ở trên nên lời giải bài toán có thể viết ngắn gọn hơn, đi thi thì học sinh nhớ chứng minh lại các ý toán đã sử dụng.

  • Gọi $T$ là giao điểm của $E F$ và $B C ; A H$ cắt $B C$ tại $D$.
  • Ta chứng minh được $T H \perp A M$ (bài 1.4), tại $Q$. Khi đó các tứ giác $H K T D, H Q M D$ nội tiếp nên
    $$
    \angle T K D=\angle T H D=\angle A M D
    $$
    suy ra $K L M D$ nội tiếp. Hơn nữa có các tứ giác $B F E C, E F D M$ nội tiếp nên
    $$
    T K \cdot T L=T D \cdot T M=T E \cdot T F=T B \cdot T C
    $$
    do đó $B K L C$ nội tiếp.

Bài 10. Cho tam giác $A B C$ nhọn, $M$ trên cạnh $B C$. Trên các cạnh $A B, A C$ lấy điểm $D, E$ sao cho $M D=M B, M E=M C$. Gọi $H$ là trực tâm tam giác $M D E$. Chứng minh rằng 4 diểm $A, D, H, E$ cùng thuộc một đường tròn.

Hướng dẫn - Gợi ý

Nhận xét: Bài này ta dễ nghĩ tới việc chứng minh $\angle D H E=180^{\circ}-\angle B A C=\angle P H Q$, trong đó $B Q, C P$ là các đường cao.
Do đó chỉ cần chứng minh tam giác $H P D$ và $P Q E$ đồng dạng, tới đây tính toán một chút vì hai tam giác này đều là các tam giác vuông.

Gọi $K, L$ lần lượt là trung điểm $B D, C E$. Ta có
$$
D P=B P-B D=B C \cos B-2 B M \cos B=\cos B(B C-2 B M)=\cos B(M C-M B)
$$
tương tự thì
$$
E Q=E C-C Q=\cos C(M C-M B)
$$

Suy ra
$$
\frac{D P}{E Q}=\frac{\cos B}{\cos C}=\frac{P B}{C Q}=\frac{H P}{H Q}
$$

Do đó
$$
\triangle H P D \backsim \triangle H Q E \Rightarrow \angle D H P=\angle E H Q, \angle D H E=\angle P H Q=180^{\circ}-\angle B A C
$$
do đó từ giác $A D H E$ nội tiếp.

Bài tập rèn luyện

Bài 1. (Chuyên Tiền Giang) Cho tam giác nhọn $A B C$ có $A B<A C$ và nội tiếp đường tròn tâm $O$. Đường tròn tâm $K$ đường kính $B C$ cắt các cạnh $A B, A C$ lần lượt tại $E, F$. Gọi $H$ là giao điểm của $B F$ và $C E$.
a) Chứng minh tam giác $A E F$ và tam giác $A C B$ dồng dạng.
b) Gọi $A^{\prime}$ là điểm đối xứng của $A$ qua $O$. Chứng minh $A A^{\prime}$ vuông góc với $E F$.
c) Từ $A$ dựng các tiếp tuyến $A M, A N$ dến đường tròn $(K)$ với $M, N$ là các tiếp điểm. Chứng minh ba điểm $M, H, N$ thẳng hàng.

Bài 2. (Chuyên Thái Nguyên) Cho tam giác nhọn $A B C$ nội tiếp đường tròn $(O), A B<A C$, các đường cao $B D, C E$ cắt nhau tại $H$ ( $D$ thuộc $A C, E$ thuộc $A B$ ). Gọi $M$ là trung điểm của $B C$, tia $M H$ cắt đường tròn $(O)$ tại $N$.
a) Chứng minh rằng năm điểm $A, D, E, H, N$ cùng nằm trên một đường tròn.
b) Lấy điểm $P$ trên đoạn $B C$ sao cho $\widehat{B H P}=\widehat{C H M}, Q$ là hình chiếu vuông góc của $A$ trên đường thẳng $H P$. Chứng minh rằng tứ giác $D E N Q$ là hình thang cân.
c) Chứng minh rằng đường tròn ngoại tiếp tam giác $M P Q$ tiếp xúc với đường tròn $(O)$.

Bài 3. (Lê Quý Đôn – Bình Định ) Cho tam giác $A B C(A B<A C)$ có các góc đều nhọn, các đường cao $A D, B E, C F$ cắt nhau tại $H$. Đường thẳng $E F$ cắt đường thẳng $B C$ và $A D$ lần lượt tại $K$ và I. Qua $F$ kẻ đường thẳng song song với $A C$ cắt $A K, A D$ lần lượt tại $M$ và $N$. Gọi $O$ là trung điểm $B C$. Chứng minh
a) $D A$ là phân giác của $\widehat{F D E}$.
b) F là trung điểm của $M N$.
c) $O D \cdot O K=O E^2$ và $B D \cdot D C=O D \cdot D K$.

Bài 4. (Chuyên TPHCM – 2018) Cho tam giác $A B C(A B<A C)$ vuông tại $A$ có đường cao $A H$. Gọi $E, F$ lần lượt là hình chiếu của $H$ lên $A B, A C$.
a) Chứng minh rằng $B E \sqrt{C H}+C F \sqrt{B H}=A H \sqrt{B C}$.
b) Gọi $D$ là điểm đối xứng của $B$ qua $H$ và gọi $O$ là trung điểm của $B C$. Đường thẳng đi qua $D$ và vuông góc với $B C$ cắt $A C$ tại $K$. Chứng minh rằng $B K$ vuông góc với $A O$.

Bài 5. (PTNK) Cho tam giác $A B C$ nhọn. Một đường tròn qua $B, C$ cắt các cạnh $A B, A C$ lần lượt tại $E$ và $F ; B F$ cắt $C E$ tại $D$. Lấy điểm $K$ sao cho tứ giác $D B K C$ là hình bình hành.
a) Chứng minh rằng $\triangle K B C$ dồng dạng với $\triangle D F E, \triangle A K C$ dồng dạng với $\triangle A D E$.
b) Hạ $D M$ vuông góc với $A B, D N$ vuông góc với $A C$. Chứng minh rằng $M N$ vuông góc với $A K$.
c) Gọi $I$ là trung điểm $A D, J$ là trung điểm $M N$. Chứng minh đường thẳng $I J$ đi qua trung điểm của cạnh $B C$.
d) Đường thẳng $I J$ cắt đường tròn ngoại tiếp tam giác $I M N$ tại $T$ (khác $I$ ). Chứng minh rằng $A D$ tiếp xúc với đường tròn ngoại tiếp tam giác $D T J$.

Đường thẳng Simson của tam giác

Bài toán 1. Cho tam giác $ABC$ nội tiếp đường tròn $(O)$ và $P$ là điểm thuộc cung $AC$ không chứa $B$. Gọi $D, E, F$ lần lượt là hình chiếu của $P$ trên các đường thẳng $BC, AC, AB$.

a) Chứng minh các tứ giác $PCDE, PDBF$ nội tiếp.

b) Chứng minh $D, P, E$ thẳng hàng.

c) Chứng minh tam giác $PDE$ và $PBA$ đồng dạng; tam giác $PFE$ và $PBC$ đồng dạng.

Lời giải.

a) Tứ giác $PCDE$ có $\angle PDC = \angle PEC = 90^\circ $ nên là tứ giác nội tiếp.

Tứ giác $PDFB $ có $\angle PDB + \angle PFB = 90^\circ + 90^\circ = 180^\circ$ nên là tứ giác nội tiếp.

b) Ta có $\angle PFE = \angle PAE$ vì $PFAE$ nội tiếp

Mà $\angle PAE = \angle PBC = \angle PFD$;

Do đó $\angle PFE = \angle PFD$, suy ra $F, E, D$ thẳng hàng.

c) Xét tam giác $PDE$ và $PBA$ có $\angle PDE = \angle PCA = \angle PBA, \angle PED = 180^\circ – \angle PCB = \angle PAB$, do đó $\triangle PDE \backsim \triangle PBA$.

Chú ý: Cho tam giác $ABC$ và $P$ là một điểm bất kì thuộc đường tròn ngoại tiếp tam giác, khi đó hình chiếu của $P$ trên các đường thẳng $BC, AC, AB$ cùng thuộc một đường thẳng. Đường thẳng này được gọi là đường thẳng Simson của điểm $P$ đối với tam giác $ABC$.

Sau đây ta xem một số bài toán liên quan đến đường thẳng simson

Bài 1. Cho tam giác $A B C$ nhọn nội tiếp đường tròn $(O), P$ là điểm thay đổi trên cung $B C$ không chứa $A$. Gọi $D, E$ là hình chiếu vuông góc của $A$ trên $P B, P C$.
a) Tìm vị trí của $P$ để $A D \cdot P B+A E \cdot P C$ lớn nhất.
b) Chứng minh rằng $D E$ đi qua một điểm cố định. Tìm vị trí của $P$ để $D E$ lớn nhất.

Hướng dẫn

a) Ta có $AD \cdot BP +AE \cdot PC = 2S_{ABP} + 2S_{ACP} = 2S_{ABPC} = 2 (S_{ABC}+S_{PBC})$

Do đó $AD \cdot BP + AE \cdot PC $ lớn nhất khi $S_{PBC}$ lớn nhất, $P$ là điểm chính giữa cung $BC$.

b) Gọi $H$ là hình chiếu của $A$ trên $BC$, khi đó $D, E, H$ thẳng hàng, hay $DE$ qua $H$ cố định.

Bài 2. Cho tam giác $A B C$, nội tiếp đường tròn $(O), P$ là điểm thuộc cung $A C$, gọi $D, E$ là hình chiếu vuông góc của $P$ trên $B C, A C$.
a) $D E$ cắt $A B$ tại $F$. Chứng minh $P F \perp A B$.
b) Gọi $M, N$ lần lượt là trung điểm $A B, D E$. Tính $\angle P N M$.

Hướng dẫn

a) Tự giải

b) Tam giác $PDE$ và $PBA$ đồng dạng, $M, N$ lần lượt là trung điểm $AB, DE$ nên $PMB$ và $PNE$ đồng dạng, suy ra $\angle PNE = \angle PMB$, từ đó $PNFM$ nội tiếp, suy ra $\agle PNM = 90^\circ$.

Bài 3. Cho tam giác $A B C$ các đường cao $A D, B E, C F$. Gọi $M, N, P, Q$ lần lượt là hình chiếu vuông góc của $D$ trên $A B, A C, B E, C F$. Chứng minh $M, N, P, Q$ thẳng hàng.

Hướng dẫn

Tứ giác $BFHD$ nội tiếp, nên hình chiếu của $D$ trên $BF, BH, FH$ thẳng hàng, hay $M, P, Q$ thẳng hàng. Tương tự cho tứ giác $CDHE$ thì $N, P, Q$ thẳng hàng.

Bài 4. Cho tam giác $A B C$ nội tiếp đường tròn $(O), P Q$ là đường kính của $(O)$.

a) Chứng minh rằng đường thẳng simson của $P, Q$ ứng với tam giác $A B C$ thì vuông góc nhau tại $I$.

b) Chứng minh $I$ thuộc đường tròn Euler của tam giác $ABC$.

Hướng dẫn

a) Xét hình như hình trên, ta có $\angle EDC = \angle EPC$ và $\angle LKB = \angle LQC$

Suy ra $\angle EDC + \angle LKB = \angle EPC + \angle LQC = 90^\circ – \angle ECP + 90^\circ – \angle QCL = 180^\circ – \angle QCP = 90^\circ$, do đó tam giác $DIK$ vuông tại $I$ hay $DE \bot LK$ tại $I$.

b) Gọi $M, N, P$ lần lượt là trung điểm $BC, AC, AB$. Do $O$ là trung điểm $PQ$ nên $M$ cũng là trung điểm $DK$, $N$ là trung điểm $LE$.

Khi đó $IN =IL = IE, IM = ID = IK$, suy ra $\angle LIN = \angle ILN = \angle CQK, \angle DIM = \angle MDK = \angle EPC$.

Do đó $\angle MIN = 90^\circ + \angle LIN + \angle DIM = 90^\circ + \angle CQK + \angle EPC = 180^\circ – \angle ACB = 180^\circ – \angle MPN$, do đó $IMPN$ nội tiếp, hay $I$ thuộc đường tròn Euler của tam giác $ABC$.

Bài 5. (IMO 2007) Xét 5 diểm $A, B, C, D, E$, sao cho $A B C D$ là hình bình hành và $B, C, D, E$ cùng thuộc một đuoòng tròn. Gọi $d$ là đuoòng thẳng qua $\mathrm{A}$, giả sủ $d$ cắt đoạn $B C$ tại $F$ và $B C$ tại $G$. Giả sủ $E F=E G=E C$, chúng minh rằng $\mathrm{d}$ là phân giác của $\angle D A B$.

Hướng dẫn

Gọi $I, H$ là trung điểm của $C G, C F$. Ta có $E I \perp C G, E H \perp C F$. Ta có $O, H, I$ thẳng hàng do $O H, O I$ cùng song song với $d$.

Dễ dàng chứng minh được $E O \perp B D$. Suy ra tam giác $E B D$ cân. Từ đó suy ra $C E$ là phân giác góc $\angle B C G$ và $d$ là phân giác $\angle D A B$.

Bài 6. Cho tứ giác $A B C D$ nội tiếp. Gọi $d_a$ là đường thẳng simson của tam giác $B C D$ ứng với điểm $A$; các đường thẳng $d_b, d_c, d_d$ xác định tương tự. Chứng minh rằng $d_a, d_b, d_c, d_d$ đồng quy.

Hướng dẫn
  • Gọi $H_a, H_b$ là trự tâm tam giác $BCD, ACD$.
  • Chứng minh $d_a$ qua trung điểm $AH_a$;
  • Chứng minh $AH_aH_bB$ là hình bình hành.

Sử dụng vectơ chứng minh các điểm thẳng hàng

Chứng minh các điểm thẳng hàng là một trong các dạng toán thường gặp trong các bài toán về vector, trong bài trình trình bày một số ví dụ, thông qua đó các em có thêm kinh nghiệm giải dạng toán này.

Tính chất 1. Cho $A, B, C$ là 3 điểm phân biệt.
a) $A, B, C$ thẳng hàng khi và chỉ khi $\overrightarrow{A B}, \overrightarrow{A C}$ cùng phương khi và chỉ khi tồn tại $k$ sao cho $\overrightarrow{A B}=k \cdot \overrightarrow{A C}$.
b) Giả sử $\overrightarrow{A B}=x \vec{a}+y \vec{b}$ và $\overrightarrow{A C}=x^{\prime} \vec{a}+y^{\prime} \vec{b}$. Khi đó $A, B, C$ thẳng hàng khi và chỉ khi tồn tại $k$ để $x=k x^{\prime}, y=k y^{\prime}$ hay $\frac{x}{x^{\prime}}=\frac{y}{y^{\prime}}$.

Tính chất 2. Cho 2 điểm $A, B$ phân biệt và điểm $O$ nằm ngoài đường thẳng $A B$. Khi đó điểm $M$ thuộc đường thẳng $A B$ khi và chỉ khi tồn tại các số $x, y$ thỏa $x+y=1$ và
$$
\overrightarrow{O M}=x \cdot \vec{a}+y \cdot \vec{b}
$$

Ví dụ 1. Cho tam giác $A B C$. Gọi $M$ là trung điểm $A B, N$ thỏa $\overrightarrow{N A}+2 \overrightarrow{N C}=\overrightarrow{0}$ và P là điểm đối xứng của B qua C.
a) Chứng minh $\overrightarrow{A N}=\frac{2}{3} \overrightarrow{A C}$
b) Chứng minh $\overrightarrow{N M}=\frac{1}{2} \overrightarrow{A B}-\frac{2}{3} \overrightarrow{A C}$.
c) Chứng minh $M, N, P$ thẳng hàng.

Lời giải

a) Ta có $\overrightarrow{0}=\overrightarrow{N A}+2 \overrightarrow{N C}=\overrightarrow{N A}+2 \overrightarrow{N A}+2 \overrightarrow{A C}=3 \overrightarrow{N A}+2 \overrightarrow{A C}$.
Suy ra $2 \overrightarrow{A C}=-3 \overrightarrow{N A}=3 \overrightarrow{A N}$.
Do đó $\overrightarrow{A N}=\frac{2}{3} \overrightarrow{A C}$.
b) Ta có $\overrightarrow{N M}=\overrightarrow{A M}-\overrightarrow{A N}=\frac{1}{2} \overrightarrow{A B}-\frac{2}{3} \overrightarrow{A C}$.
c) Ta có $\overrightarrow{P M}=\overrightarrow{B M}-\overrightarrow{B P}$
$=-\frac{1}{2} \overrightarrow{A B}-2 \overrightarrow{B C}$
$=-\frac{1}{2} \overrightarrow{A B}-2 \overrightarrow{B A}-2 \overrightarrow{A C}$
$=\frac{3}{2} \overrightarrow{A B}-2 \overrightarrow{A C}$
$=3\left(\frac{1}{2} \overrightarrow{A B}-\frac{2}{3} \overrightarrow{A C}\right)$
$=3 \overrightarrow{N M}$. Suy ra $P, M, N$ thẳng hàng.

Ví dụ 2. Cho tứ giác $A B C D$. Gọi $M, N$ thuộc cạnh $A D, B C$ sao cho $A M=2 M D, B N=2 N C$. Chứng minh rằng trung điểm các đoạn thẳng $A B, M N$ và $C D$ thẳng hàng.

Lời giải

Gọi $P, Q, R$ lần lượt là trung điểm của $A B, M N$ và $C D$.

  • Ta có $\overrightarrow{P Q}=\frac{1}{2} \overrightarrow{A M}+\frac{1}{2} \overrightarrow{B N}=\frac{1}{3} \overrightarrow{A D}+\frac{1}{3} \overrightarrow{B C}$.
  • Ta cũng có $\overrightarrow{P R}=\frac{1}{2} \overrightarrow{A D}+\frac{1}{2} \overrightarrow{B C}$.
  • Từ đó suy ra $\overrightarrow{P Q}=\frac{2}{3} \overrightarrow{P R}$, suy ra $P, Q, R$ thẳng hàng.

Ví dụ 3. Cho tam giác $A B C$ và điểm $D$ thỏa mãn $\overrightarrow{A D}=\frac{3}{4} \overrightarrow{A C}$, I là trung điểm của $B D$. M là điể thỏa $\overrightarrow{B M}=x \overrightarrow{B C}, x \in \mathbb{R}$.
a) Tinh $\overrightarrow{A I}$ theo $\overrightarrow{A B}, \overrightarrow{A C}$.
b) Tinh $\overrightarrow{A M}$ theo $x$ và $\overrightarrow{A B}, \overrightarrow{A C}$
c) Tìm $x$ để $A, I, M$ thẳng hàng.

Lời giải

a) Ta có $2 \overrightarrow{A I}=\overrightarrow{A B}+\overrightarrow{A D}=\overrightarrow{A B}+\frac{3}{4} \overrightarrow{A C}$ $\Rightarrow \overrightarrow{A I}=\frac{1}{2} \overrightarrow{A B}+\frac{3}{8} \overrightarrow{A C}$.
b) Ta có $\overrightarrow{A M}=\overrightarrow{A B}+\overrightarrow{B M}=\overrightarrow{A B}+x \overrightarrow{B C}=\overrightarrow{A B}+x(\overrightarrow{A C}-\overrightarrow{A B})=(1-x) \overrightarrow{A B}+x \overrightarrow{A C}$.
c) Ta có:
$$
\left\{\begin{array}{l}
\overrightarrow{A I}=\overrightarrow{A B}+\frac{3}{4} \overrightarrow{A C} \\\\
\overrightarrow{A M}=(1-x) \overrightarrow{A B}+x \overrightarrow{A C}
\end{array}\right.
$$

Khi đó, $A, M, I$ thẳng hàng $\Leftrightarrow \overrightarrow{A I}$ và $\overrightarrow{A M}$ cùng phương $\Leftrightarrow \frac{1-x}{1}=\frac{x}{\frac{3}{4}} \Leftrightarrow x=\frac{3}{7}$.

Bài tập rèn luyện

Bài 1. Cho tam giác $\mathrm{ABC}$. Hai điểm $\mathrm{M}, \mathrm{N}$ được xác định bởi hệ thức: $\overrightarrow{B C}+\overrightarrow{M A}=\overrightarrow{0}, \overrightarrow{A B}-$ $\overrightarrow{N A}-3 \overrightarrow{A C}=\overrightarrow{0}$. Chứng minh $M N \parallel A C$.

Bài 2. Cho $3 \overrightarrow{O A}+2 \overrightarrow{O B}-5 \overrightarrow{O C}=\overrightarrow{0}$. Chứng minh $A, B, C$ thẳng hàng.
Bài 3. Cho tam giác $A B C$ có trung tuyến $A M$. Gọi $I$ là trung điểm $A M$ và $K$ là trung điểm AC sao $A K=\frac{1}{3} A C$.
a) Biểu diễn các vectơ $\overrightarrow{B I}, \overrightarrow{B K}$ theo $\overrightarrow{B A}, \overrightarrow{B C}$.
b) Chứng minh các điểm $B, I, K$ thẳng hàng.

Bài 4. Cho tam giác $A B C$ có trọng tâm $G$. Gọi $I, J$ là hai điểm xác định bởi $\overrightarrow{I A}=2 \overrightarrow{I B}, 3 \overrightarrow{J A}+$ $2 \overrightarrow{J C}=\overrightarrow{0}$.
a) Tính $\overrightarrow{I f}, \overrightarrow{I G}$ theo $\overrightarrow{A B}, \overrightarrow{A C}$.
b) Chứng minh $I, J, G$ thẳng hàng.

Bài 5. Cho tam giác $A B C$. Lấy các điểm $M, N, P$ thỏa mãn
$$
\overrightarrow{M A}+\overrightarrow{M B}=\overrightarrow{0}, 3 \overrightarrow{A N}-2 \overrightarrow{A C}=\overrightarrow{0}, \overrightarrow{P B}=2 \overrightarrow{P C}
$$

Chứng minh $M, N, P$ thẳng hàng.

Biểu diễn vectơ theo hai vectơ không cùng phương

Tính chất 1. Cho hai vectơ $\overrightarrow{a}, \overrightarrow{b}$ khác $\overrightarrow{0}$

a) Nếu $\overrightarrow{a}, \overrightarrow{b}$ cùng phương thì tồn tại số thực $k$ sao cho $\overrightarrow{a} = k \cdot \overrightarrow{b}$.

b) Nếu $\overrightarrow{a}, \overrightarrow{b}$ không cùng phương và $ x \cdot \overrightarrow{a}+y \cdot \overrightarrow{b} = \overrightarrow{0}$, suy ra $x = y = 0$.

Chứng minh.

a) Nếu $\vec{a}, \vec{b}$ cùng phương.

  • Trường hợp 1. Nếu $\vec{a}, \vec{b}$ cùng hướng. Đặt $k=\frac{|\vec{a}|}{|\vec{b}|}$, ta chứng minh $\vec{a}=k \cdot \vec{b}$.
    Thực vậy:
    Do $k>0$ nên $k \cdot \vec{b}$ cùng hướng $\vec{b}$ mà $\vec{b}$ cùng hướng $\vec{a}$ nên $k \cdot \vec{b}$ cùng hướng $a$; Và $|k \cdot \vec{b}|=|k| \cdot|\vec{b}|=|\vec{a}|$.
  • Trường hợp 2. Nếu $\vec{a}, \vec{b}$ ngược hướng. Đặt $k=-\frac{|\vec{a}|}{|\vec{b}|}$, chứng minh tương tự như trên ta cũng có $\vec{a}=k \cdot \vec{b}$.

b) Giả sử $x \neq 0$, suy ra $\overrightarrow{a} = \dfrac{-y}{x} \cdot \overrightarrow{b}$ cùng phương $\overrightarrow{b}$, mâu thuẫn, do đó $x = 0$, dẫn đến $y = 0$.

Tính chất 2. Cho $\overrightarrow{a}, \overrightarrow{b}$ không cùng phương, khi đó với mọi vectơ $\overrightarrow{c}$ tồn tại duy nhất cặp số $(x;y)$ thỏa mãn $$\overrightarrow{c} = x \cdot \overrightarrow{a} + y \cdot \overrightarrow{b}$$

Chứng minh

  • Lấy điểm $O$ ta dựng các vectơ $\overrightarrow{A O}=\vec{a} ; \overrightarrow{O B}=\vec{b} ; \overrightarrow{O C}=\vec{c}$.
  • Từ $C$ dựng các đường thẳng song song với $O B, O A$ cắt $O A, O B$ tại $D$ và $E$. Khi đó $\overrightarrow{O C}=\overrightarrow{O D}+\overrightarrow{O E}$.
  • Mà $\overrightarrow{O D}$ và $\overrightarrow{O A}$ cùng phương nên tồn tại $x$ thỏa $\overrightarrow{O D}=x \cdot \overrightarrow{O A}=x \cdot \vec{a}$; tương tự tồn tại $y$ sao cho $\overrightarrow{O E}=y \cdot \overrightarrow{O B}=y \cdot \vec{b}$.
  • Do đó $\vec{c}=x \cdot \vec{a}+y \cdot \vec{b}$.
  • Giả sử tồn tại $x^{\prime}, y^{\prime}$ thỏa $\vec{c}=x^{\prime} \cdot \vec{a}+y^{\prime} \cdot \vec{b}$. Khi đó $x \cdot \vec{a}+y \cdot \vec{b}=x^{\prime} \cdot \vec{a}+y^{\prime} \cdot b \Leftrightarrow$ $\left(x-x^{\prime}\right) \vec{a}+\left(y-y^{\prime}\right) \vec{b}=\overrightarrow{0}$.
  • Từ tính chất 1, ta có $x = x’, y = y’$. Ta có điều cần chứng minh.

Việc biểu diễn một vec tơ theo hai vec tơ không cùng phương có nhiều ứng dụng trong việc chứng minh vec tơ bằng nhau, cùng phương, dẫn đến các bài toán chứng minh thẳng hàng, tính toán độ dài, góc, …

Ví dụ 1. Cho tam giác $A B C$ và điểm $D$ thỏa mãn $\overrightarrow{A D}=\frac{3}{4} \overrightarrow{A C}$, I là trung điểm của $B D$.
a) Tính $\overrightarrow{A I}$ theo $\overrightarrow{A B}, \overrightarrow{A C}$.
b) Cho $\overrightarrow{BM} = x \cdot \overrightarrow{BC}$. Tính $\overrightarrow{A M}$ theo $x$ và $\overrightarrow{A B}, \overrightarrow{A C}$

Lời giải.

a) Ta có $2 \overrightarrow{A I}=\overrightarrow{A B}+\overrightarrow{A D}=\overrightarrow{A B}+\frac{3}{4} \overrightarrow{A C} \Rightarrow \overrightarrow{A I}=\frac{1}{2} \overrightarrow{A B}+\frac{3}{8} \overrightarrow{A C}$.
b) Ta có $\overrightarrow{A M}=\overrightarrow{A B}+\overrightarrow{B M}=\overrightarrow{A B}+x \overrightarrow{B C}=\overrightarrow{A B}+x(\overrightarrow{A C}-\overrightarrow{A B})=(1-x) \overrightarrow{A B}+x \overrightarrow{A C}$.

Ví dụ 2. Cho tam giác $A B C$ gọi $M$ là điểm thỏa $\overrightarrow{M A}+3 \overrightarrow{M B}=\overrightarrow{0}$.
Giả sử $\overrightarrow{C M}=x \cdot \overrightarrow{C A}+y \cdot \overrightarrow{C B}$. Tính $x, y$.

Lời giải.

Ta có $\overrightarrow{0}=\overrightarrow{M A}+3 \overrightarrow{M B}=\overrightarrow{C A}-\overrightarrow{C M}+3 \overrightarrow{C B}-3 \overrightarrow{C M}$

$ \Leftrightarrow 4 \overrightarrow{C M}=\overrightarrow{C A}+3 \overrightarrow{C B} \Leftrightarrow \overrightarrow{C M}=$

$\frac{1}{4} \overrightarrow{C A}+\frac{3}{4} \overrightarrow{C B}$.

Từ đó ta có $x=\frac{1}{4}, y=\frac{3}{4}$, do sự biểu diễn $\overrightarrow{C M}$ theo $\overrightarrow{A C}, \overrightarrow{C B}$ là duy nhất.

Ví dụ 3. Cho tam giác $A B C$ và các điểm $I$, J thỏa mãn $2 \overrightarrow{C I}+3 \overrightarrow{B I}=\overrightarrow{0}, 5 \overrightarrow{J B}-2 \overrightarrow{J C}=\overrightarrow{0}$.
a) Tinh $\overrightarrow{A I}, \overrightarrow{A J}$ theo $\overrightarrow{A B}, \overrightarrow{A C}$.
b) Gọi G là trọng tâm tam giác $A B C$. Tính $\overrightarrow{A G}$ theo $\overrightarrow{A I}, \overrightarrow{A J}$.

Lời giải
Ta có:
$2 \overrightarrow{C I}+3 \overrightarrow{B I}=\overrightarrow{0} \Leftrightarrow 2 \overrightarrow{C I}+3(\overrightarrow{B C}+\overrightarrow{C I})=\overrightarrow{0} $

$\Leftrightarrow 5 \overrightarrow{C I}+3 \overrightarrow{B C}=\overrightarrow{0} \Leftrightarrow \overrightarrow{C I}=\frac{3}{5} \overrightarrow{C B} $
$ 5 \overrightarrow{J B}-2 \overrightarrow{J C}=\overrightarrow{0} \Leftrightarrow 5 \overrightarrow{J B}-2(\overrightarrow{J B}+\overrightarrow{B C})=\overrightarrow{0} $

$\Leftrightarrow 3 \overrightarrow{J B}=2 \overrightarrow{B C} \Leftrightarrow \overrightarrow{B J}=-\frac{2}{3} \overrightarrow{B C}$
a) – Tính $\overrightarrow{A I}$ theo $\overrightarrow{A B}, \overrightarrow{A C}$.
Ta có:
$$
\overrightarrow{A I}=\overrightarrow{A C}+\overrightarrow{C I}=\overrightarrow{A C}+\frac{3}{5} \overrightarrow{C B}=\overrightarrow{A C}+\frac{3}{5}(\overrightarrow{A B}-\overrightarrow{A C})=\frac{3}{5} \overrightarrow{A B}+\frac{2}{5} \overrightarrow{A C}
$$

  • Tính $\overrightarrow{A J}$ theo $\overrightarrow{A B}, \overrightarrow{A C}$.
    Ta có:
    $$
    \overrightarrow{A J}=\overrightarrow{A B}+\overrightarrow{B J}=\overrightarrow{A B}-\frac{2}{3} \overrightarrow{B C} \Leftrightarrow \overrightarrow{A B}-\frac{2}{3}(\overrightarrow{A C}-\overrightarrow{A B})=\frac{5}{3} \overrightarrow{A B}-\frac{2}{3} \overrightarrow{A C}
    $$

b) Tính $\overrightarrow{A G}$ theo $\overrightarrow{A I}, \overrightarrow{A J}$.
Đặt $\overrightarrow{A G}=x \overrightarrow{A I}+y \overrightarrow{A J}$.

$\overrightarrow{A G} =x\left(\frac{3}{5} \overrightarrow{A B}+\frac{2}{5} \overrightarrow{A C}\right)+y\left(\frac{5}{3} \overrightarrow{A B}-\frac{2}{3} \overrightarrow{A C}\right) $
$=\left(\frac{3 x}{5}+\frac{5 y}{3}\right) \overrightarrow{A B}+\left(\frac{2 x}{5}-\frac{2 y}{3}\right) \overrightarrow{A C}$

Mặt khác, $\overrightarrow{A G}=\frac{1}{3} \overrightarrow{A B}+\frac{1}{3} \overrightarrow{A C}$
$\Rightarrow \left\{\begin{array} { l }
{ \frac { 3 } { 5 } x + \frac { 5 } { 3 } y = \frac { 1 } { 3 } } \\\\
{ \frac { 2 } { 5 } x – \frac { 2 } { 3 } y = \frac { 1 } { 3 } }
\end{array} \right.$

$ \left \{\begin{array}{l}
x=\frac{35}{48} \\\\
y=-\frac{1}{16}
\end{array}\right. $

Vậy $\overrightarrow{A G}=\frac{35}{48} \overrightarrow{A I}-\frac{1}{16} \overrightarrow{A J}$

Bài tập rèn luyện

Bài 1. Cho tam giác $A B C$ và $M$ là trung điểm cạnh $B C ; N$ là điểm thuộc đoạn $A C$ sao cho $A N=2 N C$. Chứng minh rằng:
a) $\overrightarrow{A M}=\frac{1}{2}(\overrightarrow{A B}+\overrightarrow{A C})$.
b) $\overrightarrow{B N}=\frac{2}{3} \overrightarrow{A C}-\overrightarrow{A B}$
c) $\overrightarrow{M N}=\frac{1}{3} \overrightarrow{C A}-\frac{1}{2} \overrightarrow{C B}$.

Bài 2. Cho tam giác $A B C$ có $I$ là điểm đối xứng với $B$ qua $C, J$ là trung điểm $A C, K$ thuộc $A B$ thoả $A B=3 A K$.
a) Tính $\overrightarrow{B I}, \overrightarrow{B J}, \overrightarrow{B K}$ theo $\overrightarrow{B A}, \overrightarrow{B C}$.
b) Tính $\overrightarrow{I f}, \overrightarrow{I K}$ theo $\overrightarrow{B A}, \overrightarrow{B C}$.

Bài 3. Cho tam giác $A B C$. Lấy $M, N$ lần lượt là trung điểm $A B, A C$. $L$ là điểm thoả mãn $2 \overrightarrow{L A}+5 \overrightarrow{L B}+3 \overrightarrow{L C}=\overrightarrow{0}$
a) Tính $\overrightarrow{B M}, \overrightarrow{B M}, \overrightarrow{B L}$ theo $\overrightarrow{B A}, \overrightarrow{B C}$.
b) Tính $\overrightarrow{M N}, \overrightarrow{M L}$ theo $\overrightarrow{B A}, \overrightarrow{B C}$.

Tỉ lệ thức và dãy tỉ số bằng nhau – Phần 2

Bài 1. Cho $\dfrac{a}{b}=\dfrac{b}{c}=\dfrac{c}{a}$. Chứng minh rằng $a=b=c$.
Lời giải.
$$
\dfrac{a}{b}=\dfrac{b}{c}=\dfrac{c}{a}=\dfrac{a+b+c}{b+c+a}=1
$$

Khi đó, $a=b ; b=c ; c=a \Rightarrow a=b=c$.
Bài 2. Cho ba tỉ số bằng nhau là $\dfrac{a}{b+c}, \dfrac{b}{c+a}, \dfrac{c}{a+b}$. Tìm giá trị của mỗi tỉ số đó.
Lời giải.
$$
\dfrac{a}{b+c}=\dfrac{b}{c+a}=\dfrac{c}{a+b}=\dfrac{a+b+c}{2 a+2 b+2 c}=\dfrac{a+b+c}{2(a+b+c)}=\dfrac{1}{2} \text {. }
$$

Bài 3. Cho $a+b+c+d \neq 0$ và $\dfrac{a}{b+c+d}=\dfrac{b}{a+c+d}=\dfrac{c}{a+b+d}=\dfrac{d}{a+b+c}$.

Tính giá trị của: $A=\dfrac{a+b}{c+d}+\dfrac{b+c}{a+d}+\dfrac{c+d}{a+b}+\dfrac{d+a}{b+c}$.

Lời giải.
$$
\frac{a}{b+c+d}=\frac{b}{a+c+d}=\frac{c}{a+b+d}=\frac{d}{a+b+c}=\frac{a+b+c+d}{3 a+3 b+3 c+3 d}=\frac{a+b+c+d}{3(a+b+c+d)}=\frac{1}{3} \text {. }
$$

Khi đó, $3 a=b+c+d \quad ; 3 b=a+c+d \quad ; 3 c=a+b+d \quad ; 3 d=a+b+c$
$$
4 a=a+b+c+d \quad ; 4 b=a+b+c+d \quad ; 4 c=a+b+c+d \quad ; 4 d=a+b+c+d
$$

Khi đó, $4 a=4 b=4 c=4 d \Rightarrow a=b=c=d$.
Vậy $A=4$.

Bài 4. Cho tỉ lệ $\frac{a}{b}=\frac{c}{d}$. Chứng minh rằng: $\frac{a+b}{a-b}=\frac{c+d}{c-d}$.
Lời giải.
$$
\frac{a}{b}=\frac{c}{d} \Rightarrow \frac{a}{c}=\frac{b}{d}=\frac{a+b}{c+d}=\frac{a-b}{c-d}
$$

Bài 5. Cho tỉ lệ thức $\dfrac{a}{b}=\dfrac{c}{d}$. Chứng minh rằng $\dfrac{a b}{c d}=\dfrac{a^2-b^2}{c^2-d^2}$.
Lời giải.
$ \dfrac{a}{b}=\dfrac{c}{d} \Rightarrow \dfrac{a}{c}=\dfrac{b}{d}=k .$
$k^2=\dfrac{a}{c} \cdot \dfrac{b}{d}=\dfrac{a b}{c d} . $
$k^2=\dfrac{a^2}{c^2}=\dfrac{b^2}{d^2}=\dfrac{a^2-b^2}{c^2-d^2} $

Khi đó, $\dfrac{a b}{c d}=\dfrac{a^2-b^2}{c^2-d^2}$.

Bài 6. Cho $\dfrac{a}{b}=\dfrac{b}{c}=\dfrac{c}{d}$.

Chứng minh rằng: $\left(\dfrac{a+b+c}{b+c+d}\right)^3=\dfrac{a}{d}$.

Lời giải.

$\dfrac{a}{b}=\dfrac{b}{c}=\dfrac{c}{d}=\dfrac{a+b+c}{b+c+d}=k $

$\Rightarrow k^3=\left(\dfrac{a+b+c}{b+c+d}\right)^3 . $
$k^3=\dfrac{a}{b} \cdot \dfrac{b}{c} \cdot \dfrac{c}{d}=\dfrac{a}{d} $
$\left(\dfrac{a+b+c}{b+c+d}\right)^3=\dfrac{a}{d} $

Bài tập tự luyện.


Bài 1. Cho tỉ lệ thức $\frac{a}{b}=\frac{c}{d}$. Chứng minh rằng ta có các tỉ lệ thức sau: (với giả thiết các tỉ số đều có nghĩa).
(a) $\dfrac{3 a+5 b}{3 a-5 b}=\dfrac{3 c+5 d}{3 c-5 d}$;
(b) $\left(\dfrac{a+b}{c+d}\right)^2=\dfrac{a^2+b^2}{c^2+d^2}$;
(c) $\dfrac{a b}{c d}=\dfrac{(a-b)^2}{(c-d)^2}$;
(d) $\dfrac{7 a^2+5 a c}{7 a^2-5 a c}=\dfrac{7 b^2+5 b d}{7 b^2-5 b d}$.

Bài 2. Cho $\dfrac{a}{2018}=\dfrac{b}{2019}=\dfrac{c}{2020}$.

Chứng minh rằng: $ 4(a-b)(b-c)=(c-a)^2$.

Bài 3. Cho dãy tỉ số bằng nhau: $\dfrac{a_1}{a_2}=\dfrac{a_2}{a_3}=\dfrac{a_3}{a_4}=\ldots=\dfrac{a_{2018}}{a_{2019}}$.

Chứng minh rằng: Ta có đẳng thức: $\dfrac{a_1}{a_{2019}}=\left(\dfrac{a_1+a_2+a_3+\ldots+a_{2018}}{a_2+a_3+a_4+\ldots+a_{2019}}\right)^{2018}$

Tỉ lệ thức – Dãy tỉ số bằng nhau (Phần 1)

Lý Ngọc Vy – Giáo viên Star Education

Định nghĩa 1. Thương trong phép chia số $a$ cho số $b(b \neq 0)$ gọi là tỉ số của $a$ và $b$.

Định nghĩa 2. Tỉ lệ thức là đẳng thức giữa hai tỉ số.
$$
\dfrac{a}{b}=\dfrac{c}{d} \Leftrightarrow a: b=c: d
$$

Trong đó:

  • $a$ và $d$ gọi là ngoại tỉ.
  • $b$ và $c$ gọi là trung tỉ.

Tinh chất 1. Cho $a, b, c, d$ là các số khác 0 . Ta có một số tính chất sau:
(a) $\dfrac{a}{b}=\dfrac{c}{d} \Leftrightarrow a d=b c$
(b) $\dfrac{a}{b}=\dfrac{c}{d} \Leftrightarrow \dfrac{a}{c}=\dfrac{b}{d}$
(c) $\dfrac{a}{b}=\dfrac{c}{d} \Leftrightarrow \dfrac{b}{a}=\dfrac{d}{c}$.

Dãy tỉ số bằng nhau.

$$
\frac{a}{x}=\frac{b}{y}=\frac{c}{z}=\frac{a+b+c}{x+y+z}=\frac{a-b+c}{x-y+z}
$$

Với điều kiện là mẫu thức khác 0 .

Ví dụ 1. Tìm $x$ để tạo thành các tỉ lệ thức
(a) $\dfrac{3}{4}=\dfrac{x}{20}$
(b) $\dfrac{2}{2,5}=\dfrac{4}{x}$
(c) $\dfrac{3,5}{4 x}=\dfrac{5}{200}$

Lời giải.
(a) $\dfrac{3}{4}=\dfrac{x}{20} \Rightarrow x=20: 4.3 \Rightarrow x=15$.
(b) $\dfrac{2}{2,5}=\dfrac{4}{x} \Rightarrow x=4: 2.2,5 \Rightarrow x=5$.
(c) $\dfrac{3,5}{4 x}=\dfrac{5}{200} \Rightarrow x=3,5: 5.200: 4 \Rightarrow x=35$.

Ví dụ 2. Tìm $x, y$ biết:
(a) $x: y=20: 9$ và $x-y=-22$;
(b) $3 x=4 y$ và $x+2 y=35$;
(c) $x: 2=2 y: 3$ và $x y=27$;

Lời giải.
(a) $\dfrac{x}{20}=\dfrac{y}{9}=\dfrac{x-y}{20-9}=\dfrac{-22}{11}=-2 \Rightarrow x=-40 ; y=-18$.
(b) $\dfrac{x}{4}=\frac{y}{3}=\dfrac{x+2 y}{4+2.3}=\dfrac{35}{10}=\dfrac{7}{2} \Rightarrow x=14 ; y=\dfrac{21}{2}$.
(c) $\dfrac{x}{2}=\dfrac{2 y}{3}=k$
$k^2=\dfrac{x}{2} \cdot \dfrac{2 y}{3}=\dfrac{x \cdot 2 y}{6}=\dfrac{27 \cdot 2}{6}=9 \Rightarrow k=3$ hoặc $k=-3 \Rightarrow x=6 ; y=\dfrac{9}{2}$ hoặc $x=-6 ; y=\dfrac{-9}{2}$.

Ví dụ 3. Tìm $a, b$ và $c$ trong mỗi trường hợp sau:
(a) $5 a-3 b-3 c=-536$ và $\dfrac{a}{4}=\dfrac{b}{6}, \dfrac{b}{5}=\dfrac{c}{8}$;
(b) $3 a-5 b+7 c=86$ và $\dfrac{a+3}{5}=\dfrac{b-2}{3}=\dfrac{c-1}{7}$;
(c) $5 a=8 b=3 c$ và $a-2 b+c=34$;
(d) $3 a=7 b$ và $a^2-b^2=160$;

Lời giải.

(a)
$\dfrac{a}{4}=\dfrac{b}{6} \Rightarrow \frac{a}{20}=\dfrac{b}{30} ; \dfrac{b}{5}=\dfrac{c}{8} \Rightarrow \dfrac{b}{30}=\dfrac{c}{48} $
$\dfrac{a}{20}=\dfrac{b}{30}=\dfrac{c}{48}=\dfrac{5 a-3 b-3 c}{5.20-3.30-3.48}=\dfrac{-536}{-134}=4$

$\Rightarrow a=80 ; b=120 ; c=192$
(b)
$ \dfrac{a+3}{5}=\dfrac{b-2}{3}=\dfrac{c-1}{7}=\dfrac{3(a+3)-5(b-2)+7(c-1)}{3.5-5.3+7.7}=\dfrac{3 a-5 b+7 c+12}{49}=2 $
$ \Rightarrow a=7 ; b=8 ; c=15 $
(c)
$5 a=8 b=3 c \Rightarrow \dfrac{a}{\dfrac{1}{5}}=\dfrac{b}{\dfrac{1}{8}}=\dfrac{c}{\dfrac{1}{3}}$

$=\dfrac{a-2 b+c}{\dfrac{1}{5}-2 \cdot \dfrac{1}{8}+\dfrac{1}{3}}=\dfrac{34}{34}=1$

$\Rightarrow a=24 ; b=15 ; c=40 .
$
(d)
$3 a=7 b \Rightarrow \dfrac{a}{7}=\dfrac{b}{3}=k $
$k^2=\dfrac{a^2}{49}=\dfrac{b^2}{9}=\dfrac{a^2-b^2}{49-9}=\dfrac{160}{40}=4$
$\Rightarrow k=2$ hoặc $k=-2 \Rightarrow a=14$ hoặc $a=-14 ; b=6$ hoặc $b=-6$.

Ví dụ 4. Hưởng ứng phong trào quyên góp sách giáo khoa cũ giúp đỡ học sinh có hoàn cảnh khó khăn, ba lớp 7A, 7B. 7C đã quyên góp số sách tỉ lệ với $3: 4: 5$. Tính số sách giáo khoa mỗi lớp quyên góp, biết số sách quyên góp của lớp 7 hơn lớp 7A là 22 quyển.

Lời giải. Gọi $x, y, z$ lần lượt là số sách của các lớp 7A, 7B, 7C. Ta có
$$
\frac{x}{3}=\frac{y}{4}=\frac{z}{5}=\frac{z-x}{5-3}=\frac{22}{2}=11 \Rightarrow x=33 ; y=44 ; z=55
$$

Ví dụ 5. Tìm $a ; b ; c$ biết $\dfrac{12 a-15 b}{7}=\dfrac{20 c-12 a}{9}=\dfrac{15 b-20 c}{11}$ và $a+b+c=48$.
Lời giải.
$\dfrac{12 a-15 b}{7}=\dfrac{20 c-12 a}{9}=\dfrac{15 b-20 c}{11}=\dfrac{12 a-15 b-20 c+12 a-15 b+20 c}{7+9+11}=0 $
$\Rightarrow 12 a=15 b ; 20 c=12 a ; 15 b=20 c . $
$\Rightarrow \dfrac{a}{5}=\dfrac{b}{4} ; \dfrac{c}{3}=\dfrac{a}{5} ; \dfrac{b}{4}=\dfrac{c}{3} $
$\dfrac{a}{5}=\dfrac{b}{4}=\dfrac{c}{3}=\dfrac{a+b+c}{5+4+3}=\dfrac{48}{12}=4 $

$\Rightarrow a=20 ; b=16 ; c=12 $

Bài tập.

Bài 1. Tìm số hữu tỉ $x$ trong các tỉ lệ thức sau:
(a) $6: x=6,5:(-29,25)$;
(b) $14 \frac{2}{3}:\left(-80 \frac{2}{3}\right)=(0,5 . x): 35 \frac{3}{4}$;
(c) $4: x=x: 0,16$;
(d) $(1-x)^3:(-0,5625)=0,525: 0,7$;

Bài 2. Có thể lập được một tỉ lệ thức từ từng nhóm bốn số sau không?
(a) $-1 ;-3 ;-9 ; 27$;
(b) $-1 ; \frac{-1}{2} ; \frac{-1}{3} \frac{-1}{6}$;
(c) 0,$4 ; 0,04 ; 0,004 ; 0,0004$;
(d) $3^{-3} ; 3^{-5} ; 3^{-7} ; 3^{-11}$

Bài 3. Tìm $a, b, c$ biết

(a) $15 a=10 b=6 c$ và $a b c=-1920$;
(b) $a^2+3 b^2-2 c^2=-16$ và $\dfrac{a}{2}=\dfrac{b}{3}=\dfrac{c}{4}$;

Bài 4. Tìm $x ; y ; z$ biết $2 x=3 y ; 4 y=5 z$ và $4 x-3 y+5 z=7$
Bài 5. Tìm $x ; y ; z$ biết $3 x=4 y ; 5 y=6 z$ và $x y z=30$.